Vous êtes sur la page 1sur 157

SOLID

GEOMETRY

Clarita R. Guevara
Faculty of Engineering
University of Santo Tomas
Manila, 2001
FOREWORD

This book was prepared to suit the needs of engineering students taking up a two-unit course in

solid geometry. There are seven chapters covering the basic concepts of plane and solid geometry which

are needed by students preparing for the study of analytic geometry and calculus course.

The book starts with a review of terms, theorems and formulas in plane geometry which

arenecessary in visualizing and understanding the properties of three-dimensional figures. The topics

aboutsolid figures have been developed in simplified manner, beginning with the definition of terms, then

theproperties theorems, formulas and finally illustrative problems sowing the application of formulas.

Thebook puts less emphasis on the proofs of different theorems and derivation of the formulas but

moreemphasis on the properties of solid figures, their formulas for mensuration, and their applications to

theproblems using metric units. This is because of the fact that engineering is an applied science and that

thestudy of mathematics can be effective if the students solve problems themselves. The order

ofpresentation of the solid figures was done to facilitate the understanding and memorization of

thecommon solid figures.

It is hoped that the manner of presentation of the basic concepts and formulas will enable

thestudents to familiarize themselves with these basic concept; to appreciate the solid figures which

theyencounter in their everyday life; and to develop logical thinking which they need in analyzing

andsolving engineering problems.


CONTENTS

Chapter 1

Review of Plane Geometry 1

Chapter 2

Lines, Planes and Angles 29

Chapter 3

Prisms 40

Chapter 4

Cylinders 54

Chapter 5

Pyramids 63

Chapter 6

Cones 73

Chapter 7

Spheres 83
CHAPTER 1

REVIEW OF PLANE GEOMETRY

I. DEFINITION OF TERMS

A. Geometry is that branch of mathematics which deals with the properties and
relations in space of points, lines, surfaces and solids.
Plane Geometryis concerned chiefly with the study of the properties, sizes and
shapes of figures all of whose points lie in the same plane.
Solid Geometryis concerned primarily with the study of the properties, size and
shapes of figures all of whose points do not lie in the same plane.

B. Point has neither length nor width but indicates position in space. It is represented
by a dot and named by a capital letter.
Line has no width but can be extended as far as desired in either direction. It is
represented by a straight line with two arrow heads, and is named by the names of
any two of its points or by a single small letter near one end of the arrow-heads. A
straight line is the shortest distance between two points.
Plane has no thickness and it contains an infinite number of points and lines. It is
named by the name of any of its points.

C. Line Segment is a set of points which consist of two distinct points of a line and
all the points of the line between them. It has endpoints and the names of these
points give the name of the line segment. It has a definite length.
Ray is a set of points which consist of a distinct point of a line and all the points
of the line on only one side of the line. It has only one endpoint and has no
definite length. It is name by the name of any point in it.

D. Angle is a figure formed by two rays having a common endpoint. It is also


considered as the figure formed when two lines meet. It is also a figure formed
when two lines meet. It is also a figure formed by the counter-clockwise rotation
of a ray about its vertex. The vertex of an angle is the common endpoint, and its
sides are the rays forming the angle. An angle may be named by a capital letter at
the vertex when it is single angle or by the three capital letters one on each side on
the vertex of the angle (middle letter), or by using numerals or letters.
Kinds of Angles:

1. Adjacent angles are two angles having the same vertex and a common side
between them.
2. Vertical angles are two angles which have the same vertex and the sides
of one are formed by extending the sides of the other.
3. Zero angle is an angle formed by two rays with common endpoint and are
coinciding, so there is no rotation of the ray about its vertex. It is measure
0º.
4. Straight angle is an angle whose two sides from a straight line. Its measure
is 180º.
5. Right angle is one of the two equal adjacent angles formed when two lines
meet. Its measure is 90º.
6. Acute angle is an angle less than a right angle.
7. Obtuse angle is an angle greater than a right angle but less than a straight
angle.
8. Reflex angle is an angle greater than a straight angle but less two straight
angles.
9. Complementary angles are two angles whose sum measures 90º.
10. Supplementary angles are two angles whose sum measure 180º.

Congruent or equal angles are angles whose measures are equal.

E. Concurrent lines are two or more lines meeting at a point.


Perpendicular lines are two lines which intersect to form right angles.
Parallel lines are lines on the same plane that will never intersect.
Transversal is a line that intersects two or more lines at distinct points.

Note: When the transversal to two lines is drawn, eight angles are formed. The
four angles between the lines are called interior angles and the rest are the
exterior angles. The non-adjacent interior angles lying on the opposite
sides of the transversal are called alternate interior angles. While the non-
adjacent exterior angles lying in the opposite sides of the transversal are
called alternate exterior angles. Corresponding angles is a pair of non-
adjacent interior angle and exterior angle on the same side of the
transversal.

F. Midpoint of a line segment is a point that divides the line segment into two equal
parts.
Bisector of a line segment (or line) is a line that passes through the midpoint of
the line or line segment.
Angle Bisector is a ray that divides the angle into two equal angles.
Perpendicular bisector of a line is a line perpendicular to another line at the
latter’s midpoint.

G. Closed figure is a figure that begins and ends at the same point without crossing
itself.
Congruent figures are figures having the same side and shape and can be made
to coincide.
Similar figures are figures having the same shape but different sizes.

H. Polygon is a closed broken line. It is a closed figure in a plane which is the union
of segments which intersect only in their endpoints, and no two consecutive
segments are collinear.

Parts of a Polygon:

1. Sides – segments forming the polygon


2. Vertices – endpoints of the segment.
3. Angles – angles determined by only two intersecting sides specifically,
interior angles. If the angle is formed by a side and the prolongation of the
adjacent side, it is called as exterior angle.
4. Diagonal-segment whose endpoints are two non-consecutive vertices of
the polygon.
Convex polygon is a polygon whose diagonal pass through its interior. It
is a polygon each of whose angles is less than 180º.
Concave polygon is a polygon in which one or two of its diagonals do not
pass through the interior region. It is a polygon in which one or two angles
measure more than 180º.
Equilateral polygon is a polygon with congruent sides.
Equiangular polygon is a polygon with congruent angles.
Regular polygon is a polygon which is both equilateral and equiangular.

Classification of Polygons: According to number of sides.

Triangle 3 Nonagon 9

Quadrilateral 4 Decagon 10

Pentagon 5 Undecagon 11
Hexagon 6 Dodecagon (or 12
duodecagon)

Heptagon 7 Pentadecagon 15
Octagon 8 n-gon
Where n is any integer
(b) Bisector of an angle is the segment that lies on the angles of the triangle
and whose endpoints are the vertex and a point on the opposite side.
(c) Altitude is the perpendicular segment from the vertex to the opposite side
of the line containing the opposite side.
(d) Perpendicular bisector of a side is a line perpendicular to a side of a
triangle at its midpoint.

J. Quadrilateral is a polygon of four sides.

Kinds of Quadrilaterals:

Quadrilateral

General Trapezoid (with Parallelogram


quadrilateral one pair of (with two pairs of
(or trapezium) parallel sides) parallel sides)
with no two
sides parallel
Isosceles
Rectangle Rhombus (a
trapezoid
(parallelogram parallelogram
(trapezoid
with a right with two
whose non-
angle) adjacent sides
parallel sides
congruent)
are congruent)

Square (rectangle
with two adjacent
sides congruent or
a rhombus with a
right angle.

Parts of a Trapezoid:

1. Bases – parallel sides (upper and lower bases)


2. Base angles – angles including wither bases.
3. Legs – non-parallel sides
4. Median – segment joining the midpoints of the non-parallel sides.
5. Altitude – segment perpendicular to the bases or perpendicular distance between the bases.
6. Diagonal – segment joining two non-consecutive vertices.
Perimeter of a polygon is the length of the closed broken line. It is the sum of the
sum of the lengths of the sides of the polygon.

I. Triangle is a polygon of three sides.

Classification of Triangles:

According to number of congruent sides:


1. Scalene – no two sides congruent
2. Isosceles – two congruent sides
3. Equilateral – three congruent sides=

According to their angles:


1. Acute – with three acute angles
2. Right – with a right angle
3. Obtuse – with an obtuse angle

Note: equilateral triangle is a triangle whose three angles are congruent.


Isosceles right triangle is a right triangle with two congruent sides.

Parts of a Triangle:

1. Principal parts: sides, vertices, angles

Note: (a) Any side of the triangle is included by two angles if its endpoints are the
vertices of the angles.
(b) Any angle of the triangle is included by two sides if its sides are two
sides of the triangle from the same vertex.
(c) The base of a triangle is a side upon which the triangle is made to stand.
(d) For an isosceles triangle, the two congruent sides are called the legs and
the third side the base. The vertex angle is the angle included by the legs
and the base angles are the angles are the angles including the base.
(e) For the right triangle, the legs are the perpendicular sides and the third
side is called hypotenuse. The hypotenuse is the side opposite the right
angle and is the longest side.

2. Secondary parts:
(a) Median is a segment whose endpoints are a vertex of a triangle and the
midpoint of the opposite side.
K. Circle is a closed plane curve all points of which are equidistant from a point
within called the center.
Circumferenceis the length of the circle.
Radius is a line segment from the center to a point on the circle.
Chord is a line segment from the center to a point on the circle.
Diameter is a chord passing through the center of the circle.
Secant to a circle is a line with two distinct points in common with the circle.
Secant segment is a segment whose endpoints are a point on the secant but in the
exterior of the circle and the farther point of intersection with the circle.
Tangent to a circle is a straight line which has one; and only one point in common with
the circle. The common point is called the point of tangency.
Tangent segment is a segment whose endpoints are an external point on the
tangent and the point of tangency.
Common tangent is a line tangent to each of the two circles.
Common external tangent is a common tangent which is parallel to or intersects
the segment of center only when extended.
Common internal tangent is a common tangent which intersects the segment of
centers.
Segment of centers is the segment whose endpoints are the center of two circles.
Central angle is an angle formed by two radii, so its vertex is the center of the
circle and the radii are its sides.
Inscribed angle is an angle whose vertex is a point on the circle and whose sides
are chords.
Arc is any portion of the circle.
Semicircle is an arc equal to half a circle.
Major arc is arc greater than semicircle.
Minor arc is an arc less than semicircle.
An arc may be measured in the usual linear units or in arc degrees.
Arcdegree is the measure of an arc intercepted on a circle by the sides of a central
angle of 1º.
Sector of a circle is the figure bounded by two radii and the intercepted arc.
Segment of a circle is the figure bounded by an arc and its chord. A polygon
circumscribed about a circle (or circumscribed polygon) is a polygon each side
of which is tangent to a circle. The circle is said to be inscribed in the polygon.
The center of this circle is called incenter of the polygon.
A polygon inscribed in a circle (or inscribed polygon) is a polygon each side of
which is a chord of the circle. The circle is said to be circumscribed about the
polygon. The center of this circle is called the circumcenter of the polygon.
For a regular polygon, its center is the common center of the circumscribed and
inscribed circles. The radius of a regular polygon is the radius of the
circumscribed circle. The apothem of a regular polygon is the radius of the
inscribed circle. The central angle of a regular polygon is the angle formed by two
radii drawn to the extremities of any side.
Congruent or equal circles are circles whose radii are congruent.
Concentric circles are two or more circles with common center but their radii are
unequal.
Intersecting circles are circles which are tangent to a line at the same point.
Circles tangent externally are two circles whose centers are on the
opposite sides of the tangent line.
Circles tangent internally are two circles whose centers are on the same
side of the tangent line.

L. The projection of a point upon a line is foot of the perpendicular from the point to
the line.
The projection of a line segment upon a line is the locus of the projections of all the
points of the segment upon the line.
Locus of a point is the path which contains all the points and only those points which
satisfy certain given geometric conditions.

Examples: 1. the bisector of an angle is the locus of points equidistant from


the sides of the angle.
2. A circle is the locus of a point which moves in a plane so as to
be always at a fixed distance from a given fixed point.

M. Ratio of two quantities is the quotient obtained by dividing one number by another.
Proportion is the equality of two ratios. Each of the four quantities of a proportion is
a term. The first and the fourth are the extremes; the second and the third are the
means.
Mean proportion is a proportion in which the second and third terms are equal.
II. FUNDAMENTAL THEOREMS

A. Angles

1. Vertical angles are equal.


2. The sum of the angles about a point on both sides of a straight line passing
through that point is 360º.

B. Parallel Lines

1. Two lines are parallel if:


(a) They are on the same plane and perpendicular to the same line.
(b) They are lines on the same plane and parallel to the same line.
(c) They form equal alternate interior angles with a transversal.
(d) They form equal alternate exterior angles with a transversal.
(e) They form equal corresponding angles with a transversal.
(f) They form supplementary interior angles on the same side of the
transversal.

2. Through a given point only one line can be drawn parallel to given line.
3. Two parallel lines are everywhere equally distant from each other.
4. Segment of a parallel lines cut off by parallel lines are equal.

C. Triangle

1. (a) The sum of the three angles of a triangle is equal to 180º.


(b) If two sides of a triangle are equal, the angles opposite these sides are
equal; and conversely.
(c) The measure of an exterior angle of a triangle is equal to the sum of the
measures of its non-adjacent angles.
(d) If two angles of one triangle are equal respectively to two angles of another
triangle, then their third angles are equal.
(e) The bisector of an angle of triangle divides the opposite side into segments
proportional to the adjacent sides.
(f) The bisector of an exterior angle of a scalene triangle divides the opposite
side externally into segments proportional to the adjacent sides.
(g) The acute angles of a right triangle are complementary.
(h) The base angles of an isosceles triangle are equal.
2. (a) The line joining the midpoints of two sides of a triangle is parallel and equal
to one-half the length of the third side.
(b) The bisectors of the angles of a triangle are concurrent at a point equidistant
from the sides of the triangle. This point of concurrency is called the
incenter of the triangle.
(c) The perpendicular bisectors of the sides of a triangle are concurrent at a
point and equidistant from the sides of the triangle. This point of
concurrency is called the circumcenter of the triangle.
(d) The altitudes of a triangle are concurrent at a point called the orthocenter.
(e) The medians of a triangle are concurrent at a point two thirds the distance
from each vertex to the midpoint of the opposite side. This point of
concurrency is called the centroid of the triangle.

3. (a) The square of the hypotenuse of a right triangle is equal to the sum of the
squares of the legs.
(b) If in a right triangle, the altitude is drawn to the hypotenuse:
(i) The altitude is the mean proportional between the segments of the
hypotenuse.
(ii) Either leg is the mean proportional between the hypotenuse and
the projection of that leg upon the hypotenuse.

4. Two triangles are congruent if:


(a) Two sides and the included angle of one triangle are equal, respectively, to
two sides and the included angle of another (SAS).
(b) Two angles and the included side of one triangle are equal, respectively to
two angles and the included side of another (ASA).
(c) Three sides of one triangle are equal, respectively to three side of another
(SSS)
(d) Two angles and a side opposite one of them in one triangle are equal,
respectively to two angles and a side opposite one of them in another
triangle (AAS).
(e) They are congruent to the same triangle.

5. Two right triangles are congruent if:


(a) Two legs of one right triangle are equal, respectively, to two legs of
another right triangle (LL).
(b) A leg and the adjacent acute angle of one triangle are equal, respectively
to a leg and the adjacent acute angle of another right triangle (LA).

(c) The hypotenuse and a leg of one right triangle are equal respectively, to
the hypotenuse and leg of another right triangle (HL).
(d) The hypotenuse and an adjacent acute angle of another right triangle (HA).

6. Two triangles are similar if:


(a) Their corresponding angles are equal.
(b) They are similar to a third triangle.
(c) An angle of one triangle is equal to an angle of the other, and their
including sides are proportional.
(d) Their sides are respectively proportional.
(e) Their sides are respectively parallel.
(f) Their sides are respectively perpendicular.

D. Quadrilateral

1. A quadrilateral is a parallelogram if:


(a) The opposite sides are parallel.
(b) The opposite sides are equal.
(c) Two sides are equal and parallel.
(d) The diagonals bisect each other.
(e) The adjacent angles are supplementary.

2. (a) The diagonals of a rectangle are congruent.


(b) The diagonals of a rhombus are perpendicular.

3. (a) The median of a trapezoid is parallel to the bases and its length is equal to
one-half the sun of their lengths.
(b) The base angles of an isosceles trapezoid are congruent.
(c) The diagonals of an isosceles trapezoid are congruent.

4. If the adjacent mid points of the sides of quadrilateral are joined, the figure
formed is a parallelogram.

E. General Polygon

1. The sum of the interior angles of polygon of n sides is equal to (n-2) 180º
2. The sum of the exterior angles of a polygon of n sides is equal to 360 º if the
sides are produced in the same order.
3. For a regular polygon:
(a) Each interior angle is equal to 180V (n-2)/n.
(b) Each exterior angle is equal to 360 º/n, if the sides are produced in the same
order.
(c) Each central angle is equal to 360 º/n.
4. Polygons are similar if:
(a) The corresponding angles are equal.
(b) The corresponding sides are proportional.
(c) They are composed of the same number of triangles, similar each to each,
and similarly placed.
5. If two polygons are similar, then
(a) Their perimeters have the same ratio as any two corresponding sides.
(b) Their areas have the same ratio as the square of two corresponding sides.

F. Circles

1. (a) A tangent to a circle is perpendicular to the radius drawn from the point of
tangency.
(b) If two circles are tangent to each other, the line of centers passes through
the point of tangency.
(c) If two circles intersect, the line of centers is the perpendicular bisector of
their common chord.

2. In the same circle (or congruent circles), lines are congruent if:
(a) They are chords intercepted by congruent central angles.
(b) They are chords of congruent arcs.
(c) They are radii or diameters.
(d) They are chords equidistant from the center.
(e) They are perpendiculars from the center to congruent chords.
(f) They are segments of a chord formed by a radius perpendicular to it.
(g) They are tangent segments drawn to a circle from an external point.

3. In the same circle (or congruent circles), arcs are congruent if:
(a) They have equal measures.
(b) They are intercepted arcs of two congruent central angles.
(c) They are arcs subtended by two congruent chords.
(d) They are intercepted arcs of two congruent inscribed angles.
(e) They are arcs intercepted by parallel lines.

4. In the same circle (or congruent circle), angles are congruent if:
(a) They are central angles intercepting congruent arcs.
(b) They are central angles subtended by congruent chords.
(c) They are inscribed angles intercepting the same or congruent arc.
(d) They are angles made by tangents from a point to a circle with the line
from the point to the center.
5. (a) If two chords intersect in circle, the product of the segments of one is equal
to the product of the segments of the other.
(b) If two secant segments are drawn to a circle from an external point, the
product of the measures of a secant segment and its external segment is
equal to the product of the measures of the other secant segment and its
external tangent.
(c) If form an external point a secant segment and s tangent segment are drawn
to a circle, the tangent segment is the mean proportional between the secant
segment and its external segment.
(d) The perpendicular from any point on a circle to a diameter is the mean
proportional between the segments of the diameter.
(e) The chord joining any point on a circle and either endpoint of the diameter
is the mean proportional between the diameter and its projection upon the
diameter.

C
D B

A B A D
0 0

C
E

AE*EB = DE*EC AC*BC = EC*DC

A A A
C

B 0 ‫ד‬ C B C
0 0 D

D
BD AB BD AD BC AC
AB BC AD DC AC DC
6. In a circle, the measure of:
(a) A central angle is equal to the measure of the intercepted arc.
(b) An inscribed angle is equal to one-half the measure of the intercepted arc.
(c) An angle formed by a tangent and a chord is equal to one-half the measure
of the intercepted arc.
(d) An angle formed by two chords or secants intersecting within a circle is
equal to one-half the sum of the measures of the arcs intercepted by the
angle and its vertical angle.
(e) An angle formed by two secants or two tangents, or by a secant and a
tangent drawn from any external point is equal to one-half the difference
of the measures of the intercepted arcs.

A B A B
A
0 0

0
O B C

1 1
m ̂
A0B = m𝐴𝐵 m ̂
A0B =2 m𝐴𝐵 m ̂
ABC =2 m𝐴𝐵
A A
B E
0 P
0 B C
C D
1 1
m ̂ + 𝑚𝐸𝐷
ABC= 2 [𝑚𝐴𝐶 ̂] m ̂ - 𝑚𝐴𝐶
P = 2 [𝑚𝐴𝐵 ̂]

D A
A
0
P
0
P
C B
C B
1 1
m ̂ - 𝑚𝐴𝐵
P = 2 [𝑚𝐶𝐷 ̂] m ̂ - 𝑚𝐴𝐵
P = 2 [𝑚𝐴𝐶𝐵 ̂]
III. FORMULAS

A. Triangles:
Where P = perimeter
1. P = a + b + c a,b,c = sides
K = area
2. K = 1⁄2 bh b = base
h = altitude
r = apothem, radius of
3. K = 1⁄2 rP
Inscribed circle
s = semi-perimeter
4. 𝐾 = √𝑠 (𝑠 − 𝑎)(𝑠 − 𝑏)(𝑠 − 𝑐)
= 1/2 (a+b+c)

 For a right triangle:

c 𝑐 2 = 𝑎2 + 𝑏 2

K = 1⁄2 𝑎𝑏
a

 For an equilateral triangle:

P = 3e
√3
h= 𝑒
2
e e √3 2
h K= 𝑒
4

√3
r=6 𝑒
2 √3
e
R = 3ℎ = 𝑒
3

or 𝑒 = √3𝑅

Where r – apothem of the equilateral triangle


R – radius of the equilateral triangle or radius of circumscribed circle.
C. General Polygon: n-gon
Where r – radius of inscribed circle
K= 1⁄ rP
2 P - perimeter
 For a regular polygon
1 1
K = 2 𝑟𝑃 = 𝑟𝑛𝑒
2 Where r – apothem
𝑒 n - no. of sides
r= 0
2 tan(2) e – length of a sides

𝑒 0 – measure of a central angle


R= 0
2 sin( )
2

 For a regular hexagon


√3
r= 𝑒 P = 6𝑒
2

3√3 2
R=𝑒 K= 𝑒
2

D. Circles:
1. D = 2R
2. C = 2 R = D
3. K = 𝑅 2 = 1⁄4𝐷2 = 1⁄2 RC
0
4. s = 180R

5. K sector = 0 360 𝑅 2
6. K segment = K sector – K triangle
7. For two circles:
𝐾1 𝑅 2 𝐷 2 𝐶1 𝑅 𝐷
= 𝑅1 2 = 𝐷1 2 = 𝑅1 = 𝐷1
𝐾2 2 2 𝐶2 2 2

Where R = radius
D = diameter
C = circumference
K = area
S = length of arc
0 = central angle in degrees
ILLUSTRATIVE PROBLEMS

1. Each interior angle of a regular polygon is 120 and one side is 6 cm.
A) Identify the polygon
B) Find: 1) perimeter
2) Radius of the inscribed circle (apothem)
3) radius of the circumscribed circle
4) area of the inscribed circle
5) area of the circumscribed circle
6) area of the polygon
7) area of the segment of the circumscribed circle formed by one side of
the polygon

Solution:
A) For a regular polygon, 0 = 180 (n-2)/n
Substituting data, 120 = 180 (n-2)/n
Solving for n, n = 6 sides
Therefore, the polygon is a regular hexagon.

B) 1) P = 6e 5) Kc = 𝑅 2
= 6(6) = (6)2
= 36cm Kc = 113.1 𝑐𝑚2
√3
2) r = 𝑒 r = 5.2cm. 6) K = 1⁄2 rP
2

= 1⁄2(5.2)(3.6)
3) R=e K = 93.53 𝑐𝑚2
=6 cm
7) K segment = K sector – K triangle

K sector = (60/360) 𝑅 2
= 18.85𝑐𝑚2
√3
K triangle = ( 4 ) 𝑒 2
4) K1 = 𝑟 2 = 15.59 𝑐𝑚2
= (5.2)2 K segment = 18.85 – 15.59
K1 = 84.95 𝑐𝑚2 = 3.26 𝑐𝑚2
2. The base of a trapezoid are 10 cm and 22 cm, and its base angles are 60 each. If
the non-parallel sides are produced to meet, what are the areas of the triangle
formed?

Solution:
The two triangles formed are both equilateral and
similar.

√3
10 K1 = (22)2 = 209.58 𝑐𝑚2
4

By ratio and proportion:

𝐾2 𝑏2 2
=
𝐾1 𝑏1 2
60
𝐾2 (10)2
22 =
209.58 (22)2

K2 = 43.3𝑐𝑚2

3. Two chords AB and AC are drawn on a circle of radius 12 cm. find the angle
between the chords if the arc intercepted is 32cm. long.

Solution:

B C

0
0 s=
180
𝑅
0
32 =
180
 (12)

A 0 = 152.8
̂
m BOC = 0 m 𝐵𝐶
̂
m BAC = 1/2 m 𝐵𝐶
m BAC= 1/2 m (152.8)
m BAC = 76.4
Given:
̂ = 32 cm.
𝐵𝐶
̂ = 𝑂𝐶
𝑂𝐵 ̂ = 12 cm.
Name _____________________________________ Section __________________
Instructor __________________________________ Date_____________________

EXERCISES

I. Write before each number one of the three words, ALWAYS, SOMETIMES, NEVER for
each blank so that the complete statement is true. Make a sketch in support of each
answer.

_______ 1.A media of a triangle is _____________ a side of a triangle.


_______ 2.The supplement of any obtuse angle is ___________ an acute angle.
_______ 3.If two angles are complementary, then both are ____________ acute angles.
_______ 4.If a parallelogram has one right angle, then the other three angles are ____________
right angles
_______ 5.Two triangles having equal sides are _________ congruent.
_______ 6. Every tangent to a circle is ___________ perpendicular to the radius drawn to the
point of tangency.
_______ 7. A radius of a circle is _________ a diameter.
_______ 8.A quadrilateral is _____________ a four-sided figure.
_______ 9.A trapezoid is _____________ a rectangle.
_______10.A square is ____________ a rectangle.
_______11. If two chords of a circle have unequal length then the longer chord is ____________
nearer the center.
_______12. The sum of the lengths of any two sides of a triangle is _____________ equal then
the length of the third side.
_______13. If a circle is circumscribed about a triangle, then the circle ____________ contains
all the vertices of the triangle.
_______14. Corresponding angles of similar polygons are __________ equal.
_______15. An altitude of a triangle is ______________ a side of the triangle.
_______16. An obtuse triangle is _____________ a scalene triangle.
_______17. The sum of the measures of any two angles of a triangle is ___________ greater
than the measure of the third angle.
_______18. A chord of a circle is __________________ a diameter.
_______19. The vertex of a central angle is ____________ in the circle.
_______20. Any angle inscribed in a semicircle is ______________ a right angle.
_______21. If the base of an isosceles triangle is √2times as long as each of the two equal sides,
then the angle opposite the base is ______________ a right angle.
_______22. Three points of a circle are _____________ collinear.
_______23. The medians to the base of an isosceles triangle is ______________ perpendicular to
the base.
_______24. The diagonal of a rhombus are __________ perpendicular.
_______25. If a transversal intersect two lines, then the two lines are __________ parallel.
II. Solve the following:

1. Supply the missing information on the given chart.

length of
Figure Length of base(s) Area
altitude
1. triangle 8 20 ?
2. square ? ? 9
3. rectangle ? 8 72
4. parallelogram 20 ? 100
5. trapezoid 3 and ? 6 36
6. equilateral triangle ? ? 16√3

2. Given a circle 0 with radii OA and OB as pictured below. Complete the following
table.

length of area of segment


0 length of AB
radius sector AOB ABC
1. 60 ? ?  ?
2. 30° ? 4 ? ?
3. 120° 20 ? ? ?
4. ? ?   ?
5. ? 8 ?  ?

A
C
r
0 B
O

3. Complete the following table for regular polygons.


number of sides length of side Apothem
1 3 5 ?
2 3 ? √3
3 4 ? 8
4 4 √6 
5 5 5 ?
6 5 ? 1
7 6 8 ?
8 6 ? √10
9 8 2 ?
9. Find the chord of a 60 central angle in a circle of radius 20 cm.

10. The length of a diagonal of a square is 20 cm. Find the perimeter of the square.

11. A square of maximum size is cut out of a circle of radius 8cm and then a circle of
maximum size is cut out of the square. Find the area of the wasted material.
12. The area and altitude of a rectangle and a trapezoid are equal. If the base of the
rectangle is 24 cm, find the median of the trapezoid.

13. C is the center of a circle and ABCD is a rectangle with BC and AB 4 cm and 3 cm
respectively. Find the area of the shaded region.

A
B

C
D

14. The altitude of an equilateral triangle is 8cm. Find the area of the triangle.

15. Two tangent segments to a circle from an external point make an angle of 120. If
the radius of the circle is 5cm, how long are the two tangent segments?

16.

d = 10

X
60

Solve for X.
17.
C AB is a diameter of the circle
B
AB = 8, CD = 3, BD=2, CE=12, FE = 6
D
G

Find AF and CG
A
F E

18. Given a circle of diameter 30 cm, find the area of a sector and a segment whose
central angle is 150.

19. The sides of a triangle are 10cm, 12 cm and 5 cm, respectively. Find the diagonal
of a square whose area is equal to the given triangle.

20. A circle C1 is circumscribed about a regular hexagon. Another circle C2 is


inscribed in the same hexagon. If the area of C1 is 144 cm2, find the area of the
region bounded by the inscribed circle and the regular hexagon.
21. In two concentric circles, one circle has a radius thrice the length of the other.
Find the central angle that subtends the chord of one that is tangent to the other.
What is the length of this chord, if the radius of the cuter circle is 33 cm?

21. The area of an equilateral triangle is 100√3 cm2. Find the altitude of the triangle.

22. Three circles with diameters 16cm, 24cm, and 1dm, are tangent externally, each
one to the other two. Find the area of the triangle formed by joining their centers.
23. Two tangent lines drawn from an external point to a circle makes an angle of 60
with each other. The distance from the external point to the center of the circle is
10 cm. Find the radius of the circle and the length of the tangent segment.

24. The angle between a tangent and a chord is 1419’16”. If the length of the arc
subtended by the chord is 5cm, what is the radius of the circle?

25. Each two tangents from an external point to a circle is 3cm long. The smaller arc
which they intercept is 11230’26”. Find the radius of the circle.

26. The cross-section of the roof of a barn has the shape of an isosceles trapezoid
surmounted by an isosceles triangle. The bases of the trapezoid are 12m and 8m
respectively, and its altitude is 3m. The altitude of the triangle is 2m. Find the area
of the cross-section.
28. The altitude of a triangle is 30cm and its base is 80cm. Find the area of the
trapezoid formed by a line parallel to the base of the triangle and is 12cm from the
vertex.

29. The radii of two circles, with no common point are 4cm and 18cm respectively.
The portion of their common external tangent included between the points of
tangency is 22cm. Find the distance between their centers. Find the distance from
t he point of tangency of the smaller circle to the point where the line of centers
and the tangent meet.

30. A rectangular garden 9 meters long and 6 meters wide has a 1 ½ - meter path
around it. At P120.00 a square meter, find the cost to cover the path with
concrete.
CHAPTER 2

LINES PLANES AND ANGLES

Surfaces

A plane surfaces is a surface such that a straight line joining any two of its point
lies wholly in the surface.
A curved surface is a surface no part of which is a plane.

Determination of a plane

A plane is determined by:


1. Two intersecting lines.
2. A point and a line.
3. Three non-collinear point
4. Two parallel lines

Fundamental Properties of Planes

1. The intersection of a straight line and a plane is a point.


2. The intersection of two planes is a straight line.
3. An unlimited number of planes can be passed through any line.

Foot of a line

The point of intersection of a straight line and a plane is called the foot of a line.

Theorems:

A. Line Perpendicular to a Plane

1. A line is perpendicular to a plane if it’s perpendicular to every line in the


plane passing through its foot.
2. If a line is perpendicular to each of two intersecting lines at their point of
intersection, then it is perpendicular to the plane containing the two lines.
3. Through a given point in a given line there can be one plane and only one,
perpendicular to the given line.
4. Through a given point external to a given line there can be one plane and only
one perpendicular to the given line.
5. Through a given point in the plane, one line, and only one, can be drawn
perpendicular to the plane.

6. Through a given point external to a plane, one line, and only one, can be
drawn perpendicular to the plane.
7. The perpendicular is the shortest line segment from a point to a plane.
8. Two lines perpendicular to the same plane are parallel.
9. If two planes are perpendicular to each other, a line drawn in one of them
perpendicular to their intersection is perpendicular to the other.
10. If two intersecting planes are perpendicular to a third plane, their intersection
is also perpendicular to that plane.

B. Line Parallel to a Plane

1. A line and a plane are parallel if they do not meet even if extended.
2. A plane containing one and only one of two parallel lines is parallel to the
other.
3. If a line is parallel to a plane, it is parallel to the intersection of that plane
with any plane containing the line.
4. If a line and a plane are parallel, a parallel to the line through any point in
the plane lies in the plane.

C. Parallel Planes

1. Two planes are parallel if they do not meet even if extended.


2. Two planes perpendicular to the same line are parallel.
3. Through a point outside a plane, there can be one plane, and only one
parallel to a given plane.
4. If two intersecting lines are each parallel to a plane, the plane of these
lines is parallel to that plane.
5. If two angles not in the same plane have their sides parallel and extending
in the same direction, their planes are parallel.
6. The common section of a series of two or more parallel planes with
another plane is a series of two or more parallel lines.
7. If two lines are cut by a series of parallel planes, their corresponding
segments are proportional.

D. Perpendicular Planes

1. If a line is perpendicular to a given plane, every plane that contains this


line is perpendicular t the given plane.
2. Through a line not perpendicular to a plane there can be one plane, and
only one, perpendicular to the given plane.

Projection of a point on a plane

The projection of a point on a plane is the foot of the perpendicular drawn from
the point to the plane.

Projection of a line on a plane

The projection of a line on a plane is the locus of the projections of all points of
the line on the plane.

Theorems:

1. The projection on a plane of a straight line that is not perpendicular to the plane is
a straight line.
2. If a given line is not perpendicular to the given plane, the plane containing the
given line and perpendicular to the given plane cuts the given plane in a line
containing the projection of every point of the given line.

L
M
L’ M’

Skew Lines

Two non-coplanar lines which do not intersect are skew lines.

Skew Quadrilateral

A quadrilateral all of whose vertices do not lie in the same plane. It is a


quadrilateral whose opposite sides are skew.

Theorems:

1. Through two skew lines, one and only one pair of parallel planes can be drawn.
2. Through either of two skew lines there can be one plane, and only one parallel to
the other line.
3. Through a given point in space there can be one plane, and only one, parallel to
each of skew lines or else parallel to one and containing the other.
4. A line can be drawn perpendicular to both skew lines and this is the shortest
distance between the skew lines.

L M

Dihedral Angle

The union of two non-coplanar half plane angles and their common edge is called
a dihedral angle. The edge of the dihedral angle is the line of intersection of the planes.
The faces of the dihedral angles are the planes.
A dihedral angle may be read by naming a point in one face, the edge and a point
in the other face.
A plane angle of a dihedral angle is the angle formed by two rays, one in each
face, and each perpendicular to the edge of the dihedral angle at a common point.
In the figure below RST is a plane angle of the dihedral angle L-MN-O. The
plane angle of a dihedral is the measure of the dihedral angle.

N
R
L.
.O
S r

Names of Dihedral Angles

A dihedral angle is acute, right, or obtuse if its plane angle is acute, right, or
obtuse respectively.

Pairs of Dihedral Angles

Two dihedral angles are adjacent, vertical, complementary or supplementary if


their plane angles are adjacent, vertical, complementary or supplementary, respectively.

Polyhedral Angle

The union of three or more planes meeting at a common point is called polyhedral
angle. The common point is called the vertex. The planes are the faces. The intersection
of adjacent faces is the edges. The angle formed by two consecutive edges is called the
face angle.
A polyhedral angle is commonly named by the vertex and a point on each edge.
The angle pictured below is named V-LMN.
V

L
N
Names of Polyhedral Angles

A polyhedral angle having three, four, five, six faces are called trihedral,
tetrahedral, pentahedral, and hexahedral angles, respectively.

Convex Polyhedral Angle

A polyhedral angle is called convex when the section made by any plane which
cuts all the edges is a convex polygon.

Equal or Congruent Polyhedral Angles

Two polyhedral angles are congruent if the face angles and dihedral angles of one
are equal to the face angles and dihedral angles of the other. Two polyhedral angles are
equal if they can be made to coincide.

Vertical Polyhedral Angles

Vertical polyhedral angles are polyhedral angles which have a common vertex
and the edges are the prolongations of the edges of the other.

Theorems:

1. The sum of the measures of any two face angles of a trihedral angle is greater than
the measure of the third face angle.
2. The sum of the measures of the face angles of any convex polyhedral angle is less
than 360.

ILLUSTRATIVE PROBLEMS

1. What is the area of the projection on a plane of a square 10 cm on a side, if one side
of the square is parallel to the plane and another side forms an angle of 60 with the
plane?
CA’ = 10 cos 60
Solution: = 5 cm

B CA’ – is the projection of CA on


A D the plane
B’
CA = DB

DB’ – is the projection of DB on


the plane
C60
A’

Area of CA’ B’ D’ = 5 (10)


= 50 cm2

2. If two face angles of a trihedral angle are 68 and 95, between what values must the
third angle be?

Solution:
If x is third face angle and is smallest, then
X + 68> 95 or x>27
If x is the largest face angle, then
68 + 95> x or x < 163 and
X + 68 + 95< 360 or x < 197
Therefore, 27< x < 163
Name _____________________________________ Section ________________
Instructor _________________________________ Date __________________

EXERCISES

1. Find the length of the projection of a 10-m rod inclined at an angle of


(a) 45 (b) 60 (c) 30 (d) 2230’

2. A line segment 16 m long makes an angle of 70 with the plane M.


What is length of its projection?

3. The projection of a line on a plane is 4 cm. and the line is 8 cm long.


Find the angle of inclination of the line to the plane.
4. A pole 35-m long leans against a wall. Its projection is 18 m on the ground. Find the
angle of inclination of the pole to the ground.
5. Side AC of equilateral triangle ABC is perpendicular to plane M at A. If AC is 10cm
long, find the projection of AB upon plane M.

6. The base AB of a square ABCD lies in plane M and the plane of the square forms a
dihedral angle of 45 with the plane M. If AB is 10 cm, what is the length of the
projection of AC on plane M?

7. From a point 55 m up from the ground on a vertical antenna pole, three guy cables
are stretched to anchors which are located at the vertices of an equilateral triangle
20m on a side. Find the length of each cable if the foot of the pole is equidistant
from the anchors.

8. A flat roof has an inclination of 30 to the floor of a building. The roof is 50 m long
(being parallel to the floor) and 20 m wide. Find the area of the projection of the roof
upon the floor of the building.
9. What is the area of the projection on a plane of an equilateral triangle 10 cm on a
side, if one side is parallel to the plane and the other sides form an angle of 30 with
the plane?

10. A line intersects three parallel planes and its segments between the planes are 10 dm
and 15 dm. Find the distances between the planes if the line forms an angle of 60
with a line that is perpendicular to one of the planes.

11. If three parallel planes divide one line into segments of 12m and 18m, and the line
makes an angle of 60 with the first plane, find the distance between the second and
the third planes.
12. Between what limits must the third face angle of a trihedral angle lie if the other two
face angles are to be 70 and 100?

13. Can you have a trihedral angle whose face angles are (a) 60, 80, 45 (b) 20, 40,
70 (c) 120, 70, 180 (d) 65, 70 40?

14. If two angles of a trihedral angle are 130 and 140, find the lower limit which the
third face angle approaches?

15. How many kinds of polyhedral angles can be formed whose face angles are each
30? 45? 90?
Naming of polyhedrons

Polyhedrons are named according to the number of faces. A tetrahedron has four
faces, a hexahedron has six faces, an octahedron has eight faces, a dodecahedron has
twelve faces and icosahedron has twenty faces.

Convex polyhedron

The polyhedron is convex if every section of it is a convex polygon.

Regular polyhedrons

A convex polyhedron is regular if and only if its faces are congruent regular
polygons, and the same number of edges meets at its vertex.
The following are examples of regular polyhedron:

Regular tetrahedron Regular hexahedron regular octahedron

Congruent polyhedrons

Two polyhedrons are congruent if and only if pairs of corresponding faces and
pairs of corresponding polyhedral angles are congruent. These are polyhedrons which can
be made to coincide.

Similar polyhedron

Polyhedrons having the same number of faces, enclosed by polygons similar each
to each, and similarly placed and having their corresponding polyhedral angles equal.
Prisms

A prism is a polyhedron with two faces that are congruent polygons in parallel
planes and with remaining faces parallelograms with one edge contained in each of the
parallel planes.

Parts of a prism

The congruent parallel faces are called bases. The other faces are called lateral
faces. The edges of the prism not contained in either base are called lateral edges. The
altitude of the prism is the perpendicular distance between the planes of its bases.

A right section is a section made by a plane perpendicular to the lateral edges. An


oblique section is a section made by a pane oblique to the lateral edges.

altitude

lateral edge
regular prism oblique prism
lateral face

Properties of Prisms

1. The lateral faces are parallelograms.


2. The lateral edges are equal and parallel.
3. The bases are equal and parallel polygons.
4. All sections of a prism made by planes parallel to the bases and intersecting
the lateral edges are equal polygons.
5. All sections parallel to the bases are congruent to the bases.
6. All right sections are equal.
7. All sections parallel to the lateral edge is a parallelogram.
Naming of prisms

Prisms are named according to their bases. A prism with a triangular base is
called a triangular prism. A prism with a pentagonal base is a pentagonal prism.

Classification of Prisms

A right prism is a prism whose lateral edges are perpendicular to the bases. The
lateral edges are altitudes and the lateral faces are rectangles. The bases are right sections.
A regular prism is a right prism whose bases are regular polygons.
An oblique prism is a prism whose lateral edges are oblique to its bases.

Truncated Prism

A truncated prism is a polyhedron included between a base of the prism and a


section formed by a plane oblique to the base and cutting all the lateral edges.

Truncated prism

Parallelepiped

A parallelepiped is a prism whose bases are enclosed by parallelograms.

Rectangular Parallelepiped

A rectangular parallelepiped is a prism whose six faces are rectangles and the
lateral edges are perpendicular to its bases.
2. Total area of prism is the sum of the lateral area and the areas of the two bases.

AT = S + 2B where: AT – total area


B – area of the base
For a rectangular parallelepiped:

AT = 2(ab + bc + ca) where: a, b, c are the dimensions of the


rectangular parallelepiped.
For a cube:

AT = 6e2 where: e is an edge of the cube

3. Volume of a prism is the product of the area of the right section and the lateral edge.

V = eRa For a right prism: V = Bh

where: V – volume e – lateral edge B- area of the base


Ra – area of the right section h - altitude

For a rectangular parallelepiped: V = ab


For a cube: v = e3

4. Diagonal

For a rectangular parallelepiped: D = √𝑎2 + 𝑏 2 + 𝑐 2


D = 𝑒 √𝑒

Le Cavalieri’s Theorem

If two solids are included between the same two parallel planes and if the sections
made by planes parallel to the two planes are equal, then the volumes of the solids are
equal.

ILLUSTRATIVE PROBLEMS
1. Find the volume and total surface area of a rectangular hexagon prism of base edge 12
cm and altitude of 17 cm.
Solution: a) V = Bh where B = ½ rP
= ½ (12)(6)(6√3)
= 374.12 cm2

V = 374.12 (17)
V = 6360.1 cm3

b) AT = S + 2Bw
Where S = hP0
AT = 1,224 + 2(374.12) = 17 (6)(12)
= 1,224 cm2
AT = 1,972.24 cm2

2. Find the (a) diagonal, (b) volume, (c) total surface area of a rectangular parallelepiped
of dimensions 18cm by 22cm by 14cm.

Solution: a) D = √𝑎2 + 𝑏 2 + 142


D = 31.69 cm

b) V = abc
= 18 (22)(14)
= 5,544 cm3

c) AT = 2(ab + bc + ac)
= 2(18(22) + 22(14) + 18(14))
= 1,912 cm2

3. The edge of a cube is 5cm. Find (a) the total surface area (b) the face diagonal, (c) the
diagonal of the cube and (d) volume.

Solution: a) AT = 6e2
= 6(5)2
= 150 cm2

b) df = 𝑒√2
= 5√2
= 7.07 cm

c) d = 𝑒√3
= 5√3
= 8.66cm

d) V = 𝑒3
= (5)3
= 125cm3
Name _____________________________________ Section ________________
Instructor _________________________________ Date __________________

EXERCISES

1. The edge of a cube is 10cm. Find (a) total area (b) face diagonal (c) diagonal of the
cube (d) volume.

2. The lateral area of a cube is 64 cm2. Find (a) edge of the cube (b) total area (c) face
diagonal (d) diagonal of the cube (e) volume of the cube.

3. Find the volume of a cube if the sum of the areas of its faces is 150 cm2.

4. Find the total surface area of a cube whose volume is 512 cm 3.


5. Three edges which meet at one vertex of a rectangular solid are 8 cm, 10 cm and 15
cm, respectively. Find the diagonal and the total surface area of the solid.

6. The lateral edge of parallelepiped is 8 cm and a plane is passed cutting this edge at
right angle to form a right section which is a square. If the lateral area of the
parallelepiped is 320 cm2. Find the edge of the right section.

7. A plane is passed through two opposite edges of a cube forming a diagonal section
bounded by the two opposite edges and the diagonals of two opposite faces. If the
area of the section is 49√2 cm2. Find the edge of the cube.

8. A cube of edge 10 cm is cut by a plane containing two diagonally opposite edges of


the cube. Find the area of the section thus formed.

9. The length of a face diagonal of a cube is (a + b). Find its total area and volume.
10. If an edge of a cube is increased 40%, by what percentage is the volume increased?

11. Find the volume, area and length of a diagonal of a rectangular solid whose
dimensions are 10 cm, 18 cm, and 20 cm.

12. The diagonals of three faces of a rectangular solid are 10 cm, 2√34 cm and 2√41 cm.
Find the volume and total surface area of the solid.

13. Find the edge of a cube if its total area is numerically equal to three times its volume.
14. A wooden beam 16 m long contains 16 m3. If its cross-section is a square, find the
perimeter of the section.

15. The length of a rectangular parallelepiped is four times the height and the width is
three times the height. If the diagonal is √650 cm, find the volume and total area.

16. Find the total area of a right prism having a square base, if each side of the base is
6cm and the volume 540 cm3.

17. Find the volume of an oblique triangular prism whose base is an equilateral triangle
the side of which is 12 cm. The lateral edge of the prism is equal to the side of the
base and inclined to the base plane at angle 30.
18. The lateral edge of an oblique regular pentagonal prism is 8 cm. The volume is
495.5cm3. Find the edge of the right section.

19. Determine the volume and lateral area of the new prism formed from a regular
pentagonal prism by joining the consecutive midpoints of the base edges. Base edge
of the original prism was 8 cm and the altitude was 20 cm.

20. The area of a right section of an oblique prism is 525 cm2 and the volume is 7350
cm3. Find the lateral edge of the prism.

21. Find the height of a prism whose volume is 480 cm3 and whose base is a triangle with
sides 5cm, 12cm, and 13cm.
22. An edge of the base of a regular triangle prism is 9cm and the altitude of the prism is
7 cm. Find the total area of the prism.

23. The lateral area of regular pentagonal prism is 375 cm2 and the altitude of the prism is
15 cm. Find its base edge.

24. A truncated prism has a square base of edge 5 cm. Two adjacent lateral edges are 8
cm, long each and other two lateral edges are 4 cm long each. Find the total surface
area and the volume.
25. A truncated right prism has triangular lower base with each base edge 5 cm. The
lateral edges are 4 cm, 7 cm and 8 cm respectively. Find the total area.

26. A right section of an oblique prism is a rhombus whose diagonals are 10cm and 15cm
and the lateral edge of the prism is 13cm. Find the lateral area.

27. A trough whose cross-section is an isosceles right triangle is 90cm long and 50 cm
wide at the top. How many liters of water will it hold? How many liters does it
contain when the depth of the water is 15cm?

28. A dam 100m long has a cross-section which is a trapezoid whose altitude is 16 m and
whose upper base is 5 m. If the lower base angles of the cross-section are 50 and
65. Find the volume of material the dam contains.

29. How many liters of water are needed to fill a swimming pool which is 20m wide and
40m long, if when the pool is filled with water, the depth of the water increases
uniformly along the length from 1m to 3m?
CHAPTER 4

CYLINDERS

Cylindrical Surface

A cylindrical surface is a surface generated by a moving straight line which


always intersects a plane curve and remains parallel to a fixed straight line not in the
plane of the curve. The moving straight line is called the generatrix, and the curve is
called the directrix. The generatrix in any one of its position is called an element of the
surface.

A cylindrical surface may be an open cylindrical surface or a closed cylindrical


surface depending on whether the directrix is an open or closed curve.

In the figure, the generatrix, AB is P D


parallel to the fixed straight line LM. AB L B
intersects the directrix AC generating the
cylindrical surface BDCA. Line PR is an
element. E
R
A
M
Cylinder

A cylinder is a solid formed by a closed cylindrical surface and two parallel


planes cutting all the elements.

Parts of a cylinder

The parallel planes are the bases and the cylindrical surface of the lateral surface.
The altitude is the common perpendicular between the planes of the bases.
A right section of a cylinder is a section made by a plane perpendicular to one of
the elements. base

Right
section

lateral
surface

element altitude
Classification of Cylinders

A right cylinder is a cylinder in which the planes of the bases are perpendicular to one of the
elements.

An oblique cylinder is a cylinder whose elements are oblique to the base.

A circular cylinder is a cylinder whose bases are enclosed by circles. The axis of a circular cylinder
is the line joining the center of its bases.

A right elliptic cylinder is a right cylinder with ellipses as bases.

A cylinder revolution is a cylinder generated by the revolution of rectangle about one of its sides
as an axis. Hence it is a right circular cylinder.

Similar cylinders of revolution are right circular cylinders generated by the revolution of similar
rectangles about corresponding sides of the rectangles as axes.

Axis A C

A’ C’

B D B’ D’

Right circular cylinder similar cylinders of revolution

Properties of Cylinders

1. The elements are equal and parallel.


2. The bases are congruent
3. Every section of a cylinder parallel to the base is congruent to the base. For a right circular
cylinder, the sections are equal circles whose center lie in the line joining the centers of the
bases (or axis).
4. Every section of the cylinder made by a plane containing two elements is a parallelogram.
For a right circular cylinder, the section is a rectangle.
5. Any element of a right circular cylinder is equal to its altitude.

Inscribed Prism

A prism is inscribed in a cylinder when its lateral edges are elements of the cylinder and the
bases of the prism are inscribed in the bases of the cylinder.
Circumscribes Prism

A prism is circumscribed about a cylinder when the bases of the prism are circumscribed about
the bases of the cylinder and the lateral faces of the prism are tangent to the cylinder

Formulas:

For any Cylinder:

1. S = Cke
2. AT = S + 2B
3. V = Bh = Ke

Where:

S - Lateral area K – Area of right section

E – Length of element included Ck – circumference of right section


between the bases
AT – total surface are
B – Area of the base
V – Volume of the Cylinder
H – Altitude of the cylinder

For a right circular cylinder:

1. S = 2πr
2. AT = 2πrh + 2π2
3. V = πr2h
Where: t- radius of base

For similar right circular cylinders:

1. S1 = AT1 = r12 = h12 = D12

S2 AT2 r22 h22 D22

2. V1 = r13 = h13 = D13

V2 r23 h23 D23

Where D – diameter of the base subscripts 1 and 2 represents the two similar cylinders.
ILLUSTRATIVE PROBLEM

1. The height of an open cylindrical vessel that will hold 25 liters is twice the diameter. Find the total
surface area of the vessel.

V = πr2h
H = 2D = 4r
25 liters = 25000cm

H = 2D 25000 = π2 (4r)

R3 = 25000/4π

R = 12.58 cm

D H = 4(12.58) = 50.32cm

AT = S + B

=2πrh + πr2

=2π(12.58)(50.32)+π(12.58)2

=3977.42 + 497.18

=4474.60cm2

2. An element of a circular cylinder intersects the base at an angle of 60°. If the element is 12 cm and the
circumference of the base is 36 cm., find the volume of the cylinder

V = πr2h

C = 2πr

60° h 36 = 2πr

r = 5.73 cm

h = 12sin60° = 10.39cm

V = π(5.73)2 (10.39)

= 1071.78cm3
Name: Section:

Instructor: Date:

EXERCISES

1. Find the total area of a right circular cylinder having an altitude of 10 cm and a base whose radius of 8 cm.

2. Find the total area of a cylinder revolution whose altitude is 18 cm and a base whose base has a radius 12 cm.

3. The total area of cylinder of revolution is 130π m2 and the radius of the base is 5 m. find the altitude.

4. The lateral area of cylinder of revolution is 132π m2 and the circumference of the base is 12π m. find the
altitude.

5. A concrete roller is 1m long and 50 cm in diameter. What area does it cover in 500 revolution?
6. A cylinder whose altitude is 6 cm and whose total area is 572 cm2 is opened up along an element and the lateral
surface is laid out as a rectangle. Find the dimensions of the rectangle.

7. The area of the base of a right circular cylinder is 62.4 cm2 and its height is 5 cm. What is the lateral area of the
cylinder?

8. The diameter of the base of a circular cylinder is 8 cm and the elements are inclined to the base at an angle of
30°. If an element is 7 cm long, find the volume and the lateral area of the cylinder.

9. The area of the base of a cylinder is 72 cm2 , and the height is 20 cm. find the volume of the cylinder .

10. When a body is immersed in water in a right circular cylinder 40 cm in diameter, the level of the water rises 20
cm. what is the volume of the body?

11. The diameter of a well is 3cm and water is 4m deep. How many liters of waters are there in the well?

12. Find the lateral area and the total area of a cylindrical cell whose height equals its diameter if it has to hold 50
kg of water.

13. If the biggest regular triangular prism is cut from a right circular cylinder 80 cm high and 20 cm, in diameter,
what % of the volume of the cylinder is wasted?
14. Find the volume and lateral area of the cylinder that can be inscribed in a regular pentagonal prism which is
12cm on an edge and 25 cm long.

15. A regular triangular prism whose altitude is 1.5 m and each side of whose base is 2 cm is inscribed in a right
circular cylinder. Find the lateral area of the cylinder in m2.

16. A tunnel 100m long has a cross-section of a semi-circle, with the base resting on the diameter, which is 9 m in
length. If the road rests on the base and the outside walls are made of tiles. Find the number of tiles to be used
if each tile is 120cm2in area.

17. A cylindrical gasoline tank is lying on its side in a horizontal position. If the tank is 3.5 m long and 16 dm in
diameter, how many liters does it contain when the depth of the gasoline in 8 dm?

18. A cubical casting is melted in a right circular cylindrical vessel. If the total area of the casting is 937.5 cm2 and
the diameter of the vessel is 12.8 cm, how deep is the molten metal?

19. It is desired to cut off a piece of lead pipe 5 cm in outside diameter and 0.625 cm thick so that it will melt into a
cube of edge 10 cm how long a piece will be required?

20. The tin can which requires the least sheet of tin for given volume has its diameter and altitude equal. How
many such cans may be made from 10 m2 (allowing 50 cm2 per can for seams and wastes) if each can is to hold
a liter.

21. How many square meters of tin can are required to make a cylindrical bucket having a diameter of 30 cm and a
height of 40 cm, allowing 10% for waste?
22. A lead pipe 0.75 cm thick has an outer diameter of 5.5 cm. find the length of the pipe needed to weigh 10 kg if
lead weighs 11.24g/cm3.

23. The base of the right cylinder is the region shown in the figure below. The figure shown is partitioned into
sections of circles of radius 5 cm. if the altitude of the cylinder is 20 cm, find the volume and total area.

240°

240°

24. A right cylinder with an elliptical base is 20 cm high. The longer axis is thrice the length of the shorter axis. If the
volume is 192π cm3. Find the lateral area of the cylinder.
CHAPTER 5
PYRAMIDS
Pyramidal surface

A pyramidal surface generated by a straight line which moves so as always to interest a given fixed base polygon
and pass through a fixed point not in the plane of the polygon.

The moving line is called the genetrix. The directrix is the fixed polygon. The fixed point is called the vertex. The
moving line in any one of its positions is an element of the surface

A pyramidal surface consists of two part separated by the vertex each of the parts is called the nappe.

C’ B’

D’ E’ A’ APA’ is the genetrix

ABCDE is the directrix

P is the vertex

A E D

B C

Pyramid

A pyramid is a polyhedron, one of whose face is a polygon of any number of sides all the other faces are
triangles of a common vertex.

A pyramid is polyhedron formed by the nappe of closed pyramidal surface and a plane cutting all its elements.

Parts of a Pyramid

The polygonal face is called the base. The triangular faces are called the lateral faces. The intersections of the
lateral faces are called the lateral edges. The vertex is the common point of the lateral faces. The altitude is the
perpendicular from the vertex to the plane of the base.

Lateral edges

Face altitude base


Naming of Pyramids

Pyramids are named according to their bases

Triangular pyramid pentagonal pyramid hexagonal pyramid

Plane Sections of a Pyramid

1. The Section of the pyramid made by a plane parallel to the base is similar to the base.
2. The area of the section is to the area of the base as the square of its distance from the vertex is to the
square of the altitude of the pyramid.
3. If two pyramids have bases of equal areas and have equal altitudes, then the plane sections parallel to the
base and at the same distance from the vertex in each pyramid are equal in area.

𝑷𝑨 𝑷𝑩 𝑷𝑪 𝑷𝑫
k B C a) = = =
𝑷𝑳 𝑷𝑴 𝑷𝑵 𝑷𝑶

h A D N b) ABCD ~LMNO
𝑨𝒓𝒆𝒂 𝑨𝑩𝑪𝑫 𝒌^𝟐
M c) =
𝑨𝒓𝒆𝒂 𝑳𝑴𝑵𝑶 𝒉^𝟐

L O

Regular Pyramid

A regular pyramid is a pyramid whose base is a regular polygon and whose altitude is perpendicular to the base
at its center.

The lateral faces of a regular pyramid are isosceles triangles. The slant height is the altitude of any one of the
lateral faces, measured from the vertex.

Slant height

altitude
Frustum of a pyramid

The frustum of a pyramid is the part of the pyramid between the base of the pyramid and a plane section
parallel to the plane of the base.

Parts of a frustum of a pyramid


The bases of the frustum are the base of the pyramid and the section made by the plane parallel to the base.
The altitude is the perpendicular distance between the bases. The slant height of the frustum is the altitude of any
lateral faces.

lateral edges

slant height

Properties of frustum of pyramids


1. The lateral faces of frustum of a pyramid are trapezoids
2. The lateral faces of frustum of a regular pyramid are congruent isosceles trapezoids
3. The lateral edges of a frustum of a regular pyramid are equal
4. The bases of a frustum of a pyramid are similar polygon

Formulas
1. The lateral area is the summation of the areas of the lateral faces.
For a regular pyramid:

1
S= Pl Where: P – Perimeter of the base.
2

l – slant height

For a frustum of a regular pyramid:

1
S= (P+P’)l where: P – perimeter of the lower base
2

P’ – perimeter of the upper base

l - slant height

2. Volume
For pyramid: Volume is equal to one-third the product of the base and the altitude.

1
V= Bh where: h – altitude
3
B – area of the base
For a frustum of any pyramid:

Volume is equal to one-third the product of the altitude and the sum of the upper base, the lower base
and the mean proportional between the bases

1
V= h[B+B’+√𝐵𝐵′]
3

Where: h – altitude of the frustum

B – area of the lower base

B’ – area of the upper base

ILLUSTRATIVE PROBLEMS

1. Find the lateral area, volume of a regular square pyramidal all of whose edges are 5 cm in length.

Face:
l = √(𝟓)2-(2.5)

5 cm = 4.33 cm

h l Section:

h = √(4.33)2 – (2.5)2

5 cm = 3.53 cm

1 1
S= Pl V= Bh
2 3
1
= (5)(4)(4.33) = 1/3(5)2(3.53)
2
=43.3cm2 =29.46cm2
2. A plane parallel to the base of a regular square pyramid intersects the altitude at a point three-fourths the
distance from the vertex to the base. The altitude of the pyramid is 16 cm and the edge of the base is 20 cm.
find the lateral area and the volume of the frustum

Solution: Using similar triangles:


𝑥 10
=
12 16

12

X 16 4 l x=7.5cm

10 l 2.5 e1=2(7.5)=15cm

l = √(4)2+(2.5)2

= 4.717cm
1
a) S = 2(P1+P2)l

1
= (4)(20+15)(4.717)
2

= 330.19 cm2

1
b) V= [(4)(20)2+(15)2+√(20)(15)]
3

=1,233.33cm2
Name: Section:

Instructor: Date:

EXERCISES

1. An edge of the base of the regular square pyramid is 20 cm and the altitude of the pyramid is 25cm. determine
the (a) slant height (b) lateral edge (c) lateral area (d) total area (e) volume.

2. An edge of the base of a regular triangular pyramid is 12 cm.


a) Find the slant height and the total area if the altitude of the pyramid is 20cm.
b) Find the volume and the lateral area if the lateral edge is 30 cm.
c) Find the altitude and total area if the slant height is 25cm.

3. The edge of the base of a regular triangular pyramid is 10 cm and the lateral edge is 8 cm. find the lateral area
and the volume.

4. The volume of the pyramid whose altitude is 20 cm 6480 cm3. If the length of one diagonal of its rhombic base is
18cm, what is the length of the other?

5. The altitude h of a regular hexagonal pyramidal is 4 times the length of the side sof the base. What is the volume
of the pyramid in terms ofh?

6. A vessel is in the form of an inverted regular square pyramid of base of base edge 13cm and the altitude is
25cm, how many liters are in it when the depth of the water is 15cm?

7. The area of a section of a pyramid made by a plane parallel to the plane of the base is ¼ of the area of the base.
If the altitude of the pyramid is 10 cm, how far is the section from the vertex?
8. The lateral faces of a regular square pyramid are isosceles triangles whose base angles are 55° each. If the lateral
edge is 200cm, find the lateral area and volume of the pyramid.

9. A pyramid having an altitude of 450cm and a base with an area of 9302 is cut 120cm from the vertex by a plane
that is parallel to the plane of the base. Find the area of the section.

10. Find the volume and the total surface area of a frustum of a regular hexagonal pyramid if the base edges are
8cm and 6cm, respectively and the altitude is 20 cm.

11. Find the volume and the lateral area of a frustum of regular square pyramid if the base edges 4cm and 8cm and
each lateral edge is 8.5 cm.

12. The total surface area of a regular tetrahedron is 36√3 cm2. Find the volume.

13. Find the total surface area of regular tetrahedron whose altitude is 8 cm.

14. A regular octahedron is regular polyhedron composed of two square pyramids whose lateral faces is equilateral
triangles. Find the volume and the total area if a regular octahedron if an edge is 8 cm.

15. A glass paperweight has the shape of a frustum of a regular pyramid. Its height is 4cm and its upper and lower
bases are octagons whose sides are 4 cm and 3 cm, respectively. How heavy is the paper weight if the glass
weighs 2.5 grams per cubic centimeter.
16. A trashcan show in the figure has a uniform thickness of 1.5 cm and is 40 cm high. Find the capacity of the
trashcan.

25cm

25cm

18cm

18 cm

17. A pyramid was performed by connecting one vertex to the vertices of the opposite face of the cube whose edge
is 8 cm. find the volume and the total surface area of pyramid formed.

18. Given a regular solid 10 cm by 8 cm by 4 cm. find the total area of the pyramid formed, if the vertex is at the
center of the top face measuring 10 cm by 8 cm

19. A parallelepiped 8 cm by 10 cm by 12 cm, was cut by a plane passing through the midpoints of three intersecting
edges. Find the volume and total surface area of the tetrahedron formed.

20. A container has rectangular base 10 cm by 6cm. the sides of the container slope outward so the upper measures
12 cm and 8 cm. if the depth of the pan is 5 cm find the capacity of the container.
Chapter 6

Cones
Conical Surfaces

A conical surface is a surface generated by a moving straight line which constantly intersects a fixed curved
and which always passes through a fixed point not in the plane of the curve.

A C In the figure:

AB, generatrix- moving straight line

Curved EDB, directrix-fixed plane curved

V, vertex- fixed point

V CD, element-geneatrix in any one of its positions

E D B

Nappes – two portions of the conical surface on either side of the vertex.

A conical surface may be open conical surface or a closed conical surface depending on whether the directrix is
an open curve or a closed curved.

Cone

A cone is a solid bounded by a nappe of a closed conical surface and a portion of a plane intersecting all the
elements.

Parts of a Cone

1. Lateral surface – conical surface bounding the solid.


2. Base – portion of the cutting plane
3. Vertex – vertex of conical surface
4. Altitude – perpendicular distance from the vertex to the base
5. Axis – line joining the vertex and the center of the base
6. Element – part of the element of a conical surface included between the vertex and the base of the cone.
7. Right section – a section formed by a plane perpendicular to the axis and cutting all the elements.
Classification of Cones

1. Circular cone is a cone whose base is enclosed by a circle.


2. Right circular cone is a circular cone whose axis is perpendicular to the base of the cone.
3. Oblique circular cone of a circular cone whose axis is oblique to the base of the cone

*Cone of revolution is a right circular cone generated by revolving a right triangle about one of its legs as an
axis.

*Equilateral cone is a cone of revolution generated by an equilateral triangle revolving about its altitude as an axis.

*Similar Cones are cones generated by similar right triangles as they revolve about corresponding legs

altitude

Oblique Circular Cone Equilateral Cone Similar Cones

Right Circular Cone

Where:

r – radius of the circular base

h h - altitude

l - slant height (length of an element)

Properties of a right circular cone

1. The axis of the cone is also the altitude of the cone


2. The elements are equal.
3. A section containing the vertex and two points of the base is an isosceles triangle. Or it is a section
containing the axis.
4. A section parallel to the base is a circle whose center is on the axis
5. Every right section is a circle.
Conic Section

If the right circular cone is cut by a plane, the section formed is a conic section

If the cutting plane is parallel to the base, the intersection is a circle. If the plane is not parallel to the base and
cuts all the elements of the cone, but not at the vertex, the intersection is an ellipse. If the cutting plane is parallel to
one element of the cone, the intersection is a parabola. If the cutting plane intersects both nappes but does not pass
through the vertex, the intersection is a hyperbola.

Inscribed Pyramid

A pyramid inscribed in a circular cone is a pyramid all of whose lateral edges are elements of the cone and
whose base is inscribed in the base of the cone. The cone is said to be circumscribed about the pyramid.

Circumscribed Pyramid

A pyramid circumscribed about a circular cone is a pyramid all of whose lateral faces are tangent to the cone and
whose base is circumscribed about the base of the cone. The cone is said to be inscribed within the pyramid

Formulas

For a right circular cone:

1. S =πrl
2. AT=πrl+πr2
1
3. V=3πr2h

For similar cones:

1. S1/S2=AT1/At2=r12/r22=h12/h22=l12/l22
2. V1/V2=r13/r23=h13/h23=l13/l23
Frustum of a right circular cone

r where:

h r - radius of the upper base

R R – radius of the lower base

h - altitude

l – slant height

1. Lower base – base of the right circular cone


2. Upper base – section formed by the cutting plane parallel to the base of the right circular.
3. Altitude – perpendicular distance between the bases.
4. Slant height – length of that part of the element of the conical surface included between the bases of the
frustum.

Properties of a frustum of a right circular cone

1. All elements are equal


2. A section formed by a plane containing two elements is an isosceles trapezoid.

Formulas

1. S=π(r+R)l
2. AT=π(r+R)l+πr2+πR2
3. V=1/3π(r2+R2+rR)h

ILLUSTRATIVE PROBLEMS
1. A right triangle, whose legs are 16 cm and 30 cm, is revolved about the longer leg as an axis. Fint the volume and
the lateral surface of the cone generated.

Solution:

V = 1/3πr2h
30
=1/3π(16)2(30)=8042.5 cm3

16
l=√r2+h2

=√(16)2+(30)2

=34cm

S = πrl

= π(16)(34)=1709.03 cm2
2. A right circular cone of altitude 24 cm is cut by a plane parallel to the base and 15 cm from the base. Find the
ratio of the lateral surface areas of the two parts into which the cone is divided.

Solution:

Let S1 - Lateral surface area of the upper part

S2 – lateral surface area of the lower part

24 S3 – lateral surface area of the cone

15

S3/S1=h32/h12=(24)2/(9)2=64/9

S3=64/9 S1

S2=S3-S1=64/9 S1-S1=55/9 S1

Therefore:

S1/s2=9/55
Name: Section:

Instructor: Date:

EXERCISES

1. The radius of the base of a right circular cone is 6 cm and the altitude is 15 cm. determine the (a) slant height (b)
volume (c) lateral area (d) total area.

2. Find the expense, at P1800 a square meter of polishing the curved surface of a marble column with the shape of
the frustum of a right circular cone whose altitude is 4 m and the radii of whose bases are 160 cm and 85 cm
respectively

3. The total area of a right circular cone is 48π cm2. If the altitude of the cone is equal to the diameter of the base,
find the volume of the cone.

4. The sides of a right triangle are 9 cm and 15 cm.


a) If the triangle rotates about its 12-cm side and that side is fixed in position, find the total surface area and
volume of the solid generated.
b) If the triangle rotates about its 15-cm side and that side is fixed in position, find the total surface area and
volume of the solid generated.

5. A block of ice 35cm by 20 cm by 10 cm is melted in an inverted conical vessel of radius 25cm and depth of 30
cm. if the specific gravity of ice is 0.92, find the area of the wetted surface.

6. A piece of lead pipe of inner diameter 3 ½ cm, outer diameter 4 7/8 and length 1 meter has been melted in an
open conical pot of radius 15 cm and altitude 30 cm. find the depth of the molten metal.
7. A piece of paper in the form of a sector of a circle of radius 15 cm is rolled into a cone. Calculate the volume of
the cone if the angle of the sector is 240°.

8. A right circular cone of slant height 5 cm has a radius of 15 cm. find the angle of the sector of a circle of radius 10
cm, if the area of the sector is equal to the lateral area of the cone.
9. The radii of the bases of a frustum of a right circular cone are 10 cm and 8 cm, respectively and the altitude is
12cm. determine (a) slant height (b) volume (c) lateral area (d) total area.

10. The volume of a frustum of a right circular cone is 78π dm3, the upper base radius is 4 dm and lower base radius
is 7 dm. find the total area.

11. An inverted cone of revolution whose slant height of 30 cm holds a liter of liquid. How far from the vertex
should marks be placed on the slant height to indicate 250 ,l? 500ml?

12. A vessel has a shape of a frustum of a cone. The lengths of the circumferences of the frustum bases are 85 cm
and 40 cm and its height is 20 cm. find the capacity of the vessel in liter.

13. A right circular cone is inscribed in a regular tetrahedron whose edge is 10 cm. find the volume of the cone.

14. A right circular cylinder is inscribed in a right circular cone of altitude 2 m and base radius of 1 m. if the volume
of the cylinder is equal to the volume of the small cone above the cylinder, find the volume and lateral area of
the small cone.

15. A right circular cone is circumscribed about a regular triangular pyramid of height 20 dm. if the base edge of the
pyramid is 12 dm, find the volume and lateral area of the cone.

16. A funnel consists of a frustum of a right circular cone surmounted on a right circular cylinder. The funnel is 10 cm
wide at the top and is 14 cm long. The radius and altitude of the cylindrical part are 1 cm and 8 cm, respectively.
Neglecting seams and wastes, how many square meters of metal are required to make 500 funnels?
17. A tight circular cone is 10 cm high and the radius of its base is 6 cm. A plane parallel to the base divides it into
two parts whose volumes are in the ratio 3:5, the smaller part containing the vertex. Find the distance of this
plane from the base. Find also the lateral area of the frustum formed.

18. A rhombus is revolved about a side as an axis to form a solid. If each side is 12 cm and the angle between two
adjacent sided is 30°, find the volume of the solid generated.

19. Consider two cones C1 and C2. The lengths of their altitudes are h1 and h2 and the radii of their bases arer1 and r
2, respectively. Find Vc1/Vc2+ and S1/S2 if h1=4h2 and 2r1=r2.


20. If in the figure x=3, determine each of the following:
a) Ratio of the areas of circles O and Q.
b) Ratio of the volume of the cone cut off to the volume of the frustum formed.

O h

CHAPTER 7

SPHERES
Spheres

A sphere is a closed surface all points of which are equidistant from a fixed point called the center.

A radius of the sphere is a straight line segment joining the center to any point of the sphere. A diameter is a
straight line segment passing through the center and having its end points on the sphere.

A surface of a sphere is generated by the rotation of semicircle about its diameter.

Sections of a sphere

Every plane section of a sphere is a circle. A great circle of a sphere is the intersection of the sphere and a
plane passing through its center. A small circle of a sphere is the intersection of the sphere and a plane not passing a
through its center.
Axis of a circle of a sphere

The axis of a circle of a sphere is the diameter of the sphere which is perpendicular to the plane of the circle.
The extremities of the axis are called poles. Either end of any diameter of the sphere is said to be antipodal to each
other. The nearer of the two poles of a section is referred to as the pole of the section.

radius pole

great

circle axis

pole

small circle

Properties of sphere

1. All radii of a sphere are equal.


2. A point is within, on, or outside a sphere according as its distance from the center is less than, equal to, or
greater than the radius.
3. A sphere may be generated by revolving a semi-circle about its diameter as an axis.
4. The center of a great circle is center of the sphere.
5. All great circles of a sphere are equal
6. Two great circles of a sphere bisect each other.
7. 7. Sphere having equal radius are equal.

Spherical Distance

The shortest distance that can be drawn on the drawn on the surface of a sphere between two points is the
shortest arc of a great circle passing through them.

Tangent Spheres

The spheres are tangent to each other if they are tangent to a plane at the same point

Intersection of Two spheres

If two spheres intersect, their common section is a circle.


Formulas

1. The area of a sphere is equal to four times the area of one of its great circles.
Surface Area = 4πr2
Where r- radius of the sphere
2. The volume of a sphere is one-third the product of its radius and the area of its surface.

Volume=4.3πr3

Zone

A zone is a portion of the surface of a sphere included between two parallel planes.

Properties of a Zone

1. The altitude of a zone is the perpendicular between the planes.


2. If one of the planes intersects the sphere and one is tangent to the sphere, the zone is called a zone of one base
3. The bases of a zone are circles formed by the intersections of the planes and the sphere.

Area of a Zone

The area of a zone is equal to the altitude of the zone times the circumference of one great circle.

Z=2πrh

Where: r – radius of the sphere

h – altitude of the zone

Spherical Sector and Spherical Cone

Spherical sector is the sold generated by a sector of a circle about an axis which passes through the center of the
circle but which contain no point inside the sector. The bounding surfaces of a spherical sector are a zone which is called
the base, and one or two conical surfaces

A spherical sector of one conical surface is called a spherical cone.


Formulas
1. The surface area of a spherical sector is equal to the area of the zone which forms its base plus the sum of the
lateral areas bounding cones (or cones).

Total Surface Area=Zone+lateral area of bounding cones

St=Z+S1+S2 For Spherical cone S2=0

2. The volume of a spherical sector is equal to one-third of the product of the area of the zone which forms its base
and the radius of the sphere.

V=1/3Zr or V=2/3πr2h
Where: h-altitude of the zone
r-radius of the sphere

Spherical Segment
A spherical segment is a solid formed by zone and planes of its bases.

The altitude of a spherical segment is the perpendicular between the planes.

Formulas

1. The total surface area of a spherical segment is equal to the area of the zone plus the area of its bases (or base).

Total surface area=Z+A1+A2

St=2πrh+πr12+πr22

Where: r-radius of sphere

r 1 and r2-radii of the bases

2. The volume of a spherical segment is expressed by the formula:

V=1/6πh(h2+3r12+3r22

For one base:

V=1/3π2(3r-h)

Where: r-radius of the sphere


ILLUSTRATIVE PROBLEMS
1. A spherical wooden ball 48 cm in diameter sank to a depth of 28 cm when placed in water. Find the volume of
the water displaced.
Solution:
V of water displaced = V of spherical segment
= 1/3πh2(3r-h)
=1/3π(28)2 (3(24)-28)
=36,124cm3
2. A sphere is inscribed in a right circular cone of altitude 16 cm and base radius 12 cm. find the volume and
surface area of the sphere.
Solution:

By ratio and proportions:


𝑟 16−𝑟
20 cm 12
= 20
;𝑟 = 6𝑐𝑚

16-r V=4/3πr3=4/3π(6)3

=904.73cm3

A=4πr2=4π(6)2

12 =452.39 cm2

Name: Section:

Instructor: Date:

EXERCISES

1. A sphere of radius 8 cm rests in a circular hole of radius 3 cm. how far below the plane of the hole does the
sphere extend?

2. A sphere of radius 6 cm rests on 3 horizontal wires forming a plane triangle whose sides are 5 cm, 12 cm, and 13
cm. find the height of the top of the sphere above the plane of the wires.

3. A regular triangle whose area is 6√3 cm2 is inscribed in a circle of a sphere. If the plane of the triangle is 2 cm
from the nearer pole, find the area of the sphere.
4. A square is inscribed in a small circle of a sphere whose great circle has a radius of 9 cm. if the plane of the
square is 6 cm from the center of the sphere, find the perimeter of the square.

5. Two metal spheres of radii 7 cm and 10 cm, respectively are melted down and a cast into a hollow sphere of
external radius 15 cm. find the thickness of the spherical shell.

6. An ornamental spherical ball made of bronze has an outer diameter of 1.2 m and a uniform thickness of 13 cm. if
bronze weighs 8.5 gm/cm3, find the weight of the ball.

7. A hollow metal sphere 35 cm in diameter is used as a float. If the metal sphere sinks to the depth of 12 cm, what
is the area of the wetted surface?

8. On a sphere of diameter 125 cm, two circles of the sphere whose planes are parallel have radii 20 cm and 50 cm,
respectively. Find the area of the zone included between these circle.

9. The water in hemispherical bowl 80 cm across the top is 35 cm deep. Find the volume of the water in the bowl.

10. A container in the shape of a sphere of radius 5 cm is filled with water to a depth of 9 cm.
a) Find the volume of the water.
b) If the water flows out through a small hole in the bottom so that the level drops 2 cm, how much water
escaped?
11. The core of a cast iron piece has the shape of a spherical segment of two bases. The radii of the upper and lower
bases are 30 cm and 90 cm, respectively. The distance between the bases is 90 cm. find the weight of the core
assuming that sand weighs 1600 kg per cubic meter.

12. A circular sector of radius 5 cm and angle of 30° is revolved about its angle bisector. Find the volume of the solid
formed.

13. The stem of a metallic candlestick has of a spherical cone. The altitude of the conical part is 15 cm and the radius
of the base is 2 cm. find the amount of material required to make 250 stems.

14. A sphere 14 cm in diameter is inscribed in a cylinder. What is the area of the sphere? What is the lateral area of
the cylinder? What is the volume of the sphere?

15. If a sphere circumscribes a right circular cylinder of altitude 8 cm and diameter 6 cm, find the ratio of their (a)
volumes (b) total surfaces areas.

16. A sphere is placed in a cubical container in such way that the faces of the container are tangent to the sphere.
The remaining space of the container is filled with one kilogram of water. What is the radius of the sphere?

17. A rectangular parallelepiped is inscribed in a sphere whose diameter is 25 cm. find the volume of the
parallelepiped if its length is 20 cm and its width is 12 cm.

18. A conical container of altitude 8 cm circumscribed a ball 6 cm in diameter. Find the volume of the water
necessary to fill the container.
19. A metallic sphere 50 cm in radius is made into the largest right circular cone whose altitude is three times the
radius of the base. What percent of the original metal can be saved.

20. A water tank is made in the form of a right cylinder capped by hemispheres at both ends. The cylinder has a base
radius of 40 cm and an altitude of 50 cm. (a) find the total capacity of the tank. (b) if the tank contains 300 l of
water, find the area of the wetted surface.

CHAPTER 8
THEOREMS OF PAPPUS
Center of gravity
A point which a body may be supported in any position without causing the body to rotate or turn.

Ex. The center of gravity of a line segment is at the midpoint of the segment. In a square the center
of gravity is the center of the inscribed circle or the intersection of the diagonals of the square.

The First Theorem


The surface area SA of a surface of revolution generated by rotating a plane curve about an axis
external to the plane curve and on the same plane is equal to the product of the length of the generating curve x and the
distance d traveled the center of gravity of the curve.

SA=x d where: x-length of the generating curve


𝜽
SA =(x)(𝟑𝟔𝟎° (𝟐𝝅𝒓) r=distance of the center of gravity from the axis

𝜃=angle generated by the revolution of the generating curve

The Second Theorem


The volume V of solid of revolution generated by rotating a plane figure P about an external axis is equal to
the product of the area AF of the plane figure and the distance d traveled by it center gravity.

V=AFd
𝜃
V=(AF)360°(2πr)

Where : AF-area of the generating figure

r-distance of the center of gravity from the axis

𝜃-angle generated by the revolution of the generating curve.

Ex. If the length of PR is 8 cm and is 5 cm from AB, find the area of the surface of revolution generated by revolving the
line PR about the axis AB.
A F

Solution:

SA=s d
𝜽
SA=(x)(𝟑𝟔𝟎° (𝟐𝝅𝒓)

The center of gravity of PR is at its midpoint and for one complete revolution 𝜃 = 360°
360°
SA=(10)(360° (2𝜋(5)))

SA=100πcm2

Ex. Find the volume generated by revolving about the line AB the square shown in the figure.

A B

12

cm

C D

The center of gravity of the square coincides with the center of the square. The center of gravity is 6 cm from the axis
AB.
𝜃
V=(AF) (2πr)
360°

AF=area of square AF=(12)2

=144cm2
360°
V=(144)[360° (2𝜋(6))]

= 1728π cm3
Ex. Find the volume generated by revolving about the line AB the equilateral triangle as shown in the figure. An edge of
the equilateral triangle is 6 cm. A

If r is the radius of the inscribed circle of the equilateral triangle, the center of gravity is r units away from AB.

√3 √3
AF= 4 (6)2 𝜃=360° r= 6 (e)

√3
=9√3 cm2 = (6)
6

360°
V=9√3(
360°
(2𝜋[√3])) =√3

=54 cm3
MATH 122
SOLID GEOMETRY

UNIT V
PYRAMID
UNIT V
PYRAMIDS
Overview
The great pyramid of Cheops in Egypt is a classic example of the solid you will be learning in this unit.

Nowadays, structures and buildings in large cities of the world have a pyramid at their pinnacles.

This unit will help you identify the parts of a pyramid and how you obtain the information about its area and
volume. Also included in this unit is the study of its frustum.

General Objectives
At the end of this lesson, you are expected to:

1. To define a pyramid
2. Name pyramids
3. Identify the parts of the pyramid
4. Define the area and volume of pyramid
5. Define the frustum of pyramid
6. Solve pyramids and frustum of pyramid

Prerequisite Concepts
It will be to your advantage if you have a good grasp of the following:

1. Lateral development of solid


2. Compression of prism and cylinder
3. Solution of algebraic equation
What to expect
In this module, you will handle the following:

1. Pre-test to find out if you no longer have a need to go through every lesson of this module. Check your answers
using the key to the pre-test. Check the category where you belong by using this table

SCORE VERBAL INTERPRETATION


20 Excellent

18-19 Very Good

15-17 Good

12-14 Satisfactory

8-Below Needs Improvement


If you score an excellent mark, there is no need to go through this module, otherwise, you need to comprehend
every lesson in this module.

2. The lessons
a. There are three lessons in this module:
Lesson 1: what a pyramid is
Lesson 2: Solving pyramid
Lesson 3: the frustum of a regular pyramid
b. There may be three or four sections in every lesson.
These will include:
1. Specific learning objectives
2. Learning activities
3. Practice task
4. Feedback to practice task/Discussion
3. A post test is the last thing you will deal with. It will measure how much you have learned-whether stay for a
while with this module. Check your answers with the key to post test and if you got a score of 17 and above,
MOVE FORWARD!
Pre-Test
For numbers 1-3 refer to the figure.
A 1. The slant height of the pyramid is

A) AE C) AX

E D B) AY D) AB

x y 2. The height of the pyramid is

B C A) AE C) AX

B) AY D) AB

3. The lateral edge of the pyramid is

A) ED C) AX

B) AY D) AB

4. The lateral surface of the pyramid is always a

a) Square c) triangle

b) Parallelogram d) rhombus

5. For a regular hexagonal pyramid the number of lateral faces is

a) 5 c) 6

b) 7 d) 8

6. The volume of a pyramid whose height is 15 cm and whose base is a rhombus with diagonals 3 and 5 cm respectively
is

a) 75 cm3 c) 56.25 cm3

b) 37.50 cm3 d) 60 cm3

7. The base area of a regular hexagonal pyramid whose volume is 54 ft3 and whose height is 8 ft is

a) 20.25 ft2 c) 13.50 ft2

b) 10.12 ft2 d) 23.50 ft2

8. The base edge of a regular hexagonal pyramid in Prob. 7 is

a) 3.675 ft c) 3.948 ft

b) 2.280 ft d) 2.79 ft
9. The altitude of one of the lateral faces of a pyramid is

a) Lateral edge c) slant height

b) Height d) base edge

10. The slant height of a regular square pyramid whose surface area is 150 cm2 and base area is 25 cm2 is

a) 30 cm c) 18 cm

b) 15 cm d)12 cm

11. The volume of the pyramid is 6√3 in3. If the perimeter of the regular triangular base is 12 inches, the height is

a) 6 cm c) 5.5 cm

b) 4.5 cm d) 5 cm

12. The base of a pyramid is a triangle whose sides are 9 in, 12 in and 15 in respectively. If the height is 30 in, find the

volume.

a) 5400 in3 c) 540 in3

b) 810 in 2 d) 675 in3

13. Each lateral face of a frustum of a regular hexagonal pyramid is an isosceles trapezoid whose area is 12 cm2. Find the

lateral area

a)60 cm2 c) 72 cm2

b) 48 cm2 d) 50 cm2

14. A regular pentagonal pyramid whose altitude is 10 cm and base area 50 cm2 is cut by a plane parallel to the plane of

the base. Find the area of the section formed if the distance between the plane and the base is 4 cm.

a) 18 cm2 c) 8 cm2

b) 30 cm2 d) 15 cm2
15. Find the lateral area of a frustum of a regular square pyramid if the base edges are 3 cm and 6 cm respectively and

the height is 8.2 cm.

a) 145.342 cm2 c) 75.024 cm2

b) 141.714 cm2 d) 150.048 cm2

16. The base areas of a frustum a regular triangular pyramid are 4 in2 and 9 in2. Find the volume if it’s a frustum of a
regular triangular pyramid whose base altitude is 5 in.

a) 31.667 in3 c) 95 in3

b) 32.5 in2 d) 65 in3

17. The volume of a pyramid with altitude of 15 is 1521. If the length of one diagonal of its rhombic base is 24, what is
the length of the other?

a) 31.749 c) 42

b) 21 d) 36

18. The edges of a regular tetrahedron is 8 cm. the total surface area is

a) 110.85 cm2 c) 147.80 cm2

b) 221.70 cm2 d) 150.05 cm2

19. The base edge of a regular triangular pyramid is 10 cm and the slant height is 20 cm. if the pyramid is inverted and
150 cm3 of water are placed on it what is the height of the water?

a) 20.214 cm c) 17.322 cm

b) 19.792 cm d) 15.964 cm

20. The base edges of a pentagonal pyramid are each 3 cm in length. If the slant height is 10 cm, what is the height of
the pyramid?

a) 9.745 cm c) 10.211 cm

b) 4.070 cm d) 6.542 cm

Key to Pre-test

1. B 8. D 15. D
2. C 9. C 16. A
3. D 10. B 17. C
4. C 11. B 18. A
5. C 12. C 19. D
6. B 13. C 20. A
7. A 14. A
Lesson 1: what a pyramid is

I. Specific Objectives
At the end of this lesson, you are expected to:
1. Define a pyramid
2. Identify pyramids
3. Name pyramids
4. Identify the parts of the pyramids
II. Learning activities
A. Discussion

A.1 Definition of a Pyramid

These are pyramids

Are the figures polyhedrons?

Are there parallel faces in these figures?

What figure forms the lateral faces?

How many bases do these figures have?

What figure can the base assume?

Define a Pyramid

Definition of pyramid.
For your Information the triangular faces are the lateral faces meeting at a common point
called the vertex.

You now look into these pyramids and figure out they are named.

Square Triangular Hexagonal


octagonal

How do you name pyramids?

A.2 The parts of a pyramid

There are two types of pyramid given below. It is now time to analyze the parts of a
pyramid

A A

D C

E B

E 0 C Irregular pyramid

Regular pyramid

AB, AC are lateral edges.

What is a lateral edge?

Can you name the other lateral edges?


ED is a base edge.

What is a base edge?

What are the other base edges?

A is the vertex of the vertex pyramid. What is the vertex of a pyramid?

AE and AO are the altitudes of the pyramids. What is the altitude of the pyramid?
In the irregular pyramid, is the vertex above the center of pyramid? What about the vertex of a
regular pyramid?

Define a regular pyramid

Definition of a regular pyramid

For your information, the altitudes of the lateral face of a regular pyramid are known as the
slant height of the pyramid.

III. Feedback to discussion

A.1

Yes, they are polyhedrons.

None. There are no parallel faces in the figures.

A triangle forms the lateral face.

They only have one base.

The base may assume any polygon.

A pyramid is a polyhedron bounded by triangular planes meeting at a point and a plane


cutting through the triangular planes.

Pyramids are named after their bases.

A.2

A lateral edge is the intersection between two lateral faces.

AD and AE are the other lateral edges.

A base edge is any side in the base of the pyramid.

EB, BC and DC are the other base edges.


The vertex is the point where the triangular faces of the pyramid meet.

The altitude of the pyramid is the measure of the perpendicular distance from the
vertex to the plane of the base.

No, the vertex is not above the center of the base.

In a regular pyramid, the vertex is directly above the center of the base.

A regular pyramid whose base is a regular polygon and whose vertex is above the center
of the base.
A.2

A lateral edge is the intersection between two lateral faces.

AD and AE are the other lateral edges.

A base edge is any side in the base of the pyramid.

EB, BC, and DC are the other base edges.

The vertex is the point where the triangular faces of the pyramid meet.

The altitude of the pyramid is the measure of the perpendicular distance from the
vertex to the plane of the base.

No, the vertex is not above the center of the base.

In a regular pyramid, the vertex is directly above the base.

A regular pyramid is a pyramid whose base is a regular polygon and whose vertex
is above the center of the base.
Lesson 2: Solving Pyramids

I. Specific Objectives

At the end of this discussion, you are expected to:


1. Define the areas and volume of a pyramid
2. Solve areas and volume of a pyramid

II. Learning Activities


A. Discussion

A.1 the LATERAT and TOTAL AREA of a Pyramid

Consider the irregular pyramid

A A A A A

B D

C B C D E B

D C

How do you get the lateral area? ____________________________

How do you get the total area? ______________________________


You now consider a regular pyramid of a square base.

A A A A A

e e e

e e

What can you say about lateral faces of a regular pyramid? __________________

What does the altitude of a lateral face correspond to? ___________________

What is the area of one lateral face? ___________________________________

How many lateral faces are there? _____________________________________

What can you say about the base edge and the base of each lateral face?
__________________________________

Express the surface area in terms of the base edge and slant height
___________________________

If there is n base edge, what is the surface are? ________________________

What is n times b? __________________________

Write the lateral area as a function of perimeter and slant height.

Lateral Area of a Regular pyramid

S= _____________________________________
Write the total area.

Total Area of a Regular Pyramid

T.A = ___________________________

B. Illustrative examples

1. The base edges of a hexagonal pyramid are 3 cm each in length. If the


height of the lateral face is 10 cm, find a) lateral edge; b) height of the
pyramid; c) lateral area; d) total area.

Solution.

10 Le h e

10
1.5 cm r

3 cm lateral face section

a) Consider the lateral face, lateral edge, le is:


Le= √ (10)2 + (1.5)2
=10.112 cm
b) A section of the pyramid is drawn by a passing plane through the vertex
and the slant height of the pyramid. Using Pythagorean Theorem,

h = √e2 – r2

Where r = √3e, the apothem of a regular hexagon

Substituting e=3, then r = 2.598 cm.

Solving for the value of h, h= √102 – 2.5982

Therefore, h = 9.656 cm.

c) The lateral area, S, is 6 time the area of one isosceles triangle.


Thus, S = 6[1∕2 (3) (10)]
= 90 cm2

Also, S = 1∕2 Pbe

= 1∕2 (6)(3)(10)

= 90 cm2

d) The total area is T.A. = S+B


B is the area of the regular hexagon base.
3√3
Recall the unit I the Kregular hexagon= 𝑒^2
2
3√3
Thus, B= 2
3^2 = 23.383 cm2 and
T.A. = 90 + 23.383
= 113.383 cm2
2. Find the lateral area and the total area of a regular triangular pyramid of a
base edge 10 cm of 15 cm.

e
15 cm

10 cm

Consider a section of the pyramid which involves the slant height, e, and the
apothem r.

The value of r is the apothem of an equilateral triangle.

√3
From the Unit I, you have learned this to be 𝑒
6

Substituting e = 10 cm. yields r = 20887 cm.

Using Pythagorean Theorem

e = √152 + 2.8872

= 15.275 cm.

Thus, the lateral area is S = 1∕2 (3)(10)(15.275)

= 229.128 cm2
The total are is T.A. = S + B

In this exercise, you have to recall from Unit 1 the area of a regular triangle, i.e.,
√3
Kequilateral triangle = 𝑒^2 3
2

Substituting the value e = 10cm, then B = 41.310 cm2

Then, T.A. = 229.128 + 13.301

= 272.429 cm2

3. If the lateral edge of a regular 8√3 cm and makes an angle of 60°with the
plane of the base, find the lateral area and the total area.

Solution.

8√3 cm

Consider a section of the pyramid involving the lateral edge, le = 8√3 cm, and the
radius, R, of the square. From the section you see that
𝑅
Cos60° =
8√3

R = 8√3 cos60°
h = 6.928 cm
Recall from Unit 1, for squares, e = √2R. Solving for the measure of the base edge,

e = √2 (6.928)

= 9.797 cm

You now consider the lateral face of the pyramid. The slant height may now be
obtained.

e = √ le2 – r2

=√(8√3)2 – ((1∕2)9.797)2
e le = 12.96 cm

The lateral area is s = 1∕2Pbe

= 1∕2(4)(9.797)(12.96)

= 253.971 cm

The lateral area is T.A. = S+B. the base is a square so B= e2.

Thus, T.A. = 253.971 + (9.797)2

= 349.952 cm2

A.2 The volume of a Pyramid


To obtain the volume of a pyramid you have to start with a prism. Consider a
triangular prism.
E A A A E

A
D
D
B F F

F C

B C C F

Are AB and EF equal? ______________________________

What are they called relative to the pyramids? ___________________________

Are BCF and ADE equal? _________________________________

Do pyramids A-BEF and F-ADE have the same volumes? _____________________

Are DCF and DEF equal? ___________________________

Do pyramids A-DCF and A-DEF have the same volumes? _____________________

Are the volumes of the three pyramids A-BCF, A-DCF and A-DEF equal to the
volume of the prism?__________________________

What is the volume of the pyramid relative to the volumes of a prism?


___________________________

Recall, that the volume of a prism is B times h. write the volume of the pyramid in
terms of a volume of a prism

Volume of a Pyramid

V = ________________________________
B. Illustrative Examples
1. The base of the pyramid is a regular hexagon, each of whose side is 8 cm. if
the slant height is 10 cm. what is the volume of the pyramid?

Solution.

You first solve the area of the base, B.

For the regular hexagon,


3√3
B= 𝑒^2
2

3√3
l=10 cm = (8)^2 h 10
2

= 166.27 cm2

e=8 cm r

We need to solve first the apothem, r, before we can solve the height. The
apothem of a regular hexagon is
√3
r= 𝑒
6

Substituting e= 8 cm, r= 2.309 cm

You now apply Pythagorean Theorem in the section containing the apothem
and the slant height.

h= √(10)2 – (2.309)2

= 9.729 cm
The volume is V= 1/3Bh. Substituting the values of B and h yields,

V= 1/3(166.27)(9.729)

=539.256 cm3

2. A square pyramid has its vertex directly opposite one of its vertices as
shown. Find the volume.

Solution.

8 You know that the base is a square and its altitude is 8 units

5 long.

5 Thus, V = 1/3Bh

= 1/3 (5)2(8)

= 66.67 cubic units.

3. Find the volume of a regular triangular pyramid if its lateral area is 210 cm2
and each base edge is 7 cm.

h l
7 cm

√3
r= e
6

Solution:

To be able to solve for the volume, you need to find the height of the pyramid. To do
that, you have to deal with a section of the pyramid involving the slant height, apothem and the
altitude.

The lateral area is S = 1/2Pbl. The base is an equilateral triangle, so Pb =3(e). Since e = 7
cm., then Pb = 21 cm.
Thus, 210 = ½(21)l and l = 20 cm.

Consider now the section. If you apply Pythagorean Theorem, you can solve for
the height, h.

√3
h = √202 - ( (7))^2
6

= 19.897 cm.
√3
The base will have an area equal to (7)^2. The base area is
4

B = 21.218 cm2.

The volume of the pyramid is now determined.

V = 1/3 (21.218) (19.897)

= 140.722cm3

Practice Task

They say that the skeleton of a regular tetrahedron when mounted on


one’s head becomes a very good source of energy to fire up one’s mental
capabilities. After going through the concepts of a pyramid, you should be a more
informed and a better learned individual. You will only conclude that is true when
you have solved these exercises:
a) Prob. 2 p.66 g.) Prob. 7 p.67
b) Prob. 3 p.66 h.) Prob. 6 p.67
c) Prob. 5 p.67 i.) Prob. 17 p.69
d) Prob.12 p.68 j.) Prob. 18 p.69
e) Prob. 13 p. 68 k.) Prob.19 p.69
f) Prob. 14 p.68

The answers are found on the Feedback to Practice Task. You should be
extra careful in dealing with pyramids. PATIENCE is what you need. Do not
be in such a hurry. Take your time!

III. Feedback to Practice Task/Discussion


A.1

The lateral area is obtained by getting all the areas of the triangular faces.

The total area is the sum of the lateral and the base area.

The lateral faces of a regular pyramid are congruent triangles.

The altitude of a lateral face corresponds to the slant height of the pyramid.

The area of a lateral face is ½(e) (l ); where e is the measure of the base
edge and l is the slant height of the pyramid.

There are four lateral faces.


The base edge and the base of the lateral edge are equal.

S = 4[1/2(e) (l)]

S = n [1/2 (e) (l)]

The product ne is the perimeter of the base, Pb.

S = ½ Pbl

T.A = S + B

Prob.2) Ex.a) 30.199 cm.; 605.936 cm2

Ex b) 389.961 cm3; 346.423 cm2

Ex c) 24.759 cm.; 512.354 cm3

Prob.3) 93.675 cm2; 79.919 cm3


√3
Prob.5) h
32

Prob.12) 25.455 cm3

Prob.13) 140.354 cm2

Prob.14) 58.917 cm3; 86.6 cm2

Probl.7) 5 cm.

Probl.6) 0.3402 L.

Prob.17) 170.667 cm3; 218.512 cm2

Probl.18) 187.794 cm2

Prob.19) 20 cm3; 58.656 cm2


Lesson 3: Frustum of a Regular Pyramid

I. Specific Objectives
At the end of this lesson, you are expected to:
1. Define the frustum of a pyramid
2. Define the area and volume of the frustum of a pyramid
3. Solve the frustum of a pyramid

II. Learning Activities


A. Discussion

A.1 Definition of a FRUSTU of a PYRAMID

To produce the frustum of a pyramid, pass a cutting plane parallel to the base.
There are two solids produced. One is a pyramid and the other is the frustum of
the pyramid.
Describe the frustum.

___________________________________________________________________
___________________________________________________________________

If the pyramid is regular, what happens to the lateral faces?


_______________________________________

Define the frustum of a Pyramid

Definition of the Frustum of a Pyramid

For your information, the altitude of the frustum is the perpendicular distance
between the bases. Whereas the slant height of the frustum is the altitude of the
trapezoidal face.

A.2 the VOLUME of the FRUSTUM of a Pyramid

Relate the volume of the frustum to the volumes of the pyramids.


__________________________________

What is the volume of the big pyramid? ____________________________

What is the volume of the small pyramid? __________________________

Write the volume of the frustum in terms of the above volume.


_______________________________________________________
What is the altitude of the original pyramid in terms of the altitudes of the small
pyramid and the frustum? ___________________________________________

Simplify the resulting equation. ____________________________________

You must be able to express y, the altitude of the small pyramid in terms of h, the
altitude of the frustum. To do that, you’ll have to make use of the theorem stated
below.
The area of the section is to the area of the base as the square of its distance
from the vertex is to the square of the altitude of the pyramid.

Using the theorem, solve for y in terms of h and the bases of the pyramids.
_____________________________________________________

Don’t forget to rationalize. _________________________________

Substitute y in the simplified equation and solve for the volume.

Volume of a Frustum of a Pyramid

V = _____________________________
A.3 the LATERAL and TOTAL AREA OF THE frustum OF A PYRAMID

B2

H= y B1
H
B2
B1

ORIGINAL PYRAMID SMALL PYRAMID FRUSTUM OF PYRAMID

What will make up the lateral area? _______________________________

Are the lateral faces congruent? ___________________________________

What is the area of a lateral face? __________________________________

How many faces are there? ________________________________________


what is the lateral area? __________________________________________

If there are n numbers of the base edges, what will be the surface area?
_________________________________________

What is the n times sum of base edges of the trapezoid?


_____________________________

Write the surface area in terms of the perimeters of the bases and the slant
height of the frustum.

Lateral Area of a Frustum of a Pyramid

V = _____________________________
What is the total area?

Total Area of a Frustum of a Pyramid

T.A. = ______________________________

B. Illustrative Examples

1. The base edges of a frustum of a regular square pyramid are 12 inches and
10 inches respectively, and the height of the frustum is 10 inches. Find the
a) lateral area b) total area and c) volume.

Solution:

10 in

To be able to solve for the areas, 10

You need to get the slant height

of the frustum.

12 in
Consider the section containing the altitude of the slant height.

5 in

10 in
l

5 in 1 in

6 in

a) S = ½ 1(P1 +P2) where P1 = 4(12) = 48 in


P2 = 4(10) =40 in

The slant height l is obtained by the use of Pythagorean Theorem.

l = √(10)2 – (1 )2

= 9.95 in

Thus, the lateral area is s = ½ (9.95)(48+40)

= 437.8 in2

b) The total area is T.A = S + B1 + B2

B1 is the lower base area and B2 is the upper base area.

Thus, T.A. = 437.8+ (10)2+(12)2

= 681.8 in2
c) V = 1/3h[B1+B2+√B1B2 ]
= 1/3(10)[(10)2+(12)2 ]
= 1213.333 in2
2. A regular pentagonal pyramid with base area of 10 ft2 and height of 7 ft is
cut by the plane 3ft from the vertex. Find the area of the section formed.
Applying the theorem on page 28,
Area section = h2
3 Area base H2
B = 32
7 10 72
B= 1.84 ft2

Practice Task

You have now reached the last stage of your study of pyramids. After going
through the activities if this lesson, you will be faced by a greater challenge
yet very fulfilling when you have overcome the exercises of your textbook:

a.) Prob. 10 p. 68
b.) Prob. 11 p. 68
c.) Prob. 15 p. 69
d.) Prob. 16 p. 69
e.) Prob. 20 p. 39
How did you come up with the challenge? Check out with the Feedback to
Practice Task. If you have been victorious, there must not be any fear in dealing
with exercises of the Post Test. Cheers!

III. Feedback to Practice Task/Discussion

A.1

The frustum of a pyramid is bounded by trapezoidal faces and two similar


and parallel polygons similar to the base of the pyramid.

The lateral faces become congruent lateral faces.

The frustum of a pyramid is a portion of the pyramid and has two parallel
bases which are similar polygons. The lateral faces take the form of trapezoid.

A.2

V frustum = V big pyramid = V small pyramid

1/3B1H

1/3B2y

1/3 B1H - 1/3B2y

Y+h

1/3B1H - 1/3B2y

1/3B1(y+h) - 1/3B2y or 1/3[(B1 - B2)y + B1h


𝐵1 𝑦
B2 = y2 Square both sides: √ =
𝐵2 𝑦+ℎ

B1 (y+h)2
√B1 − B2 𝑦
Rationalizing, = . Manipulate the equation and arrive at the
B1 𝑦+ℎ
ℎ√𝐵1𝐵2+𝐵2
expression for y. that is, y =
𝐵1−√𝐵1𝐵2
ℎ[√𝐵1𝐵2+𝐵2
Rationalizing the resulting expression, you obtain y =
𝐵1−𝐵2
(𝐵1−𝐵2)ℎ√𝐵1𝐵2
Substituting in the simplified equation yields 1/3[ (𝐵1−𝐵2)
+ 𝐵1ℎ]
Simplifying, the volume of the frustum of the pyramid is
1/3[B1+B2+√B1 B2]h

A.3

The lateral area will be made up of the areas of the trapezoidal faces.

Yes, they are congruent

The area of an isosceles trapezoid is ½ (b1 + b2) l

There are four faces.

4 1/2(b1 + b2) l = ½(4b1 + 4b2) l

n 1/2(b1 + b2) l = ½(nb1 + nb2) l

These are the perimeters of the bases of the frustum.

S = ½(P1 + P2)l

The total area is the sum of lateral area and the base areas.

T.A. = S + B1 + B2
Prob. 10) 658.18 cm3; 439.59 cm2

Prob. 11) 211.062 cm3; 165.22 cm2

Prob. 15) 595.453 gm

Prob. 16) 383.17 cm3

Prob. 15 595.453 cm3

Post Test
1. The dimensions of the great Pyramid of Egypt were approximately 750ft
along each side is the square base and a height of a 481ft. What is the
surface area?
a) 1,590,990.25ft2 c) 721,859ft2
b) 360,750ft2 d) 914,859ft2
2. Compare the area of a regular tetrahedron every edge of which measures 2
meters with the area of a regular octahedron each of whose edge is also 2
meters. (Note: a regular octahedron consists of two square pyramids joined
at their bases.)
a) 1:2 c) 1:2√3
b) 1:4 d) √3 :2
3. A regular hexagonal pyramid has a altitude of 5cm and every base edge
measures 4 cm. find the volume.
a) 10√3 cm3 c) 40√3 cm3
b) 20√3 cm3 d) 80√3 cm3
4. What is the volume of a pyramid whose altitude is 12ft whose base has 19
sides with an area of 23 square ft?
a) 38 ft3 c) 276 ft3
b) 92 ft3 d) 437 ft3
5. What is the edge of the cube whose volume is equal to regular tetrahedron
whose sides is 8 inches?
a) 4.2 in c) 3.1 in
b) 3 in d) 3.9 in
6. What is the volume of a pyramid whose base is triangle with sides 26 cm,
28 cm, and 15 cm and whose altitude is 10 cm?
a) 625 cm3 c) 1867.72 cm3
b) 642.65 cm3 d) 1870 cn3
7. A pyramid of altitude 12 meters has a rhombus for its base. The diagonals
are 6 m and 8.4 m. find the volume.
a) 302.4 m3 c) 201.6 m3
b) 151.2 m3 d) 100.8 m3
8. What part of the volume of a parallelepiped is the volume of the pyramid
whose lateral edges are the three edges of a parallelepiped meeting at a
point?
a) ½ c) 1/4
b) 1/3 d) 1/6
9. A concrete volume is in the form of a frustum of a regular pyramid having a
square base. The column is 15 ft high, and the side of the bases is 4 ft and 2
ft respectively. Find the weight of the column if one cubic foot of concrete
weighs 7045 kilograms.
a) 670.5 kg c) 1080.25 kg
b) 1043 kg d) 1788 kg
10.A pyramid 9ft high has a base area of 20 sq. ft. The pyramid is cut by a
plane parallel to the plane of the base 3 ft from the vertex. What is the
volume of the frustum?
a) 144 ft3 c) 171 ft3
b) 234 ft3 d) 342 ft3
11.Find the volume of the frustum of a regular triangular pyramid having sides
of 8 inches and 12 inches respectively and an altitude of 10 inches.
a) 196.3 in3 c) 438.8 in3
b) 337.8 in3 d) 1013.3 in3
12.Find the volume of a regular quadrangular pyramid if a lateral edge is 101
cm and the base edge is 40 cm
a) 5,171.2 cm3 c) 53,866.7 cm3
b) 5,386.6 cm3 d)m51,712 cm3
13.A regular pyramid whose base is a regular hexagon6 cm on a side and
whose altitude is 30 cm, is cut by a plane parallel to the plane of the base
and 20 cm from its vertex. Find the lateral area of the frustum formed.
a) 295.5 cm2 c) 304.5 cm2
b) 300 cm2 d) 416.5 cm2
14. The area of a section of pyramid made by a plane parallel to the plane of
the base is ¼ of the area of the base. If the section of the pyramid is 5 cm
from the vertex, how high is the pyramid?
a) 2 1/2 cm c) 12 cm
b) 10 cm d) 15 cm
15.The base area of a frustum of a triangular pyramid is 4 in2 and 9 in2
respectively and the altitude is 5 in. find the volume.
a) 32 ½ in3 c) 31 2/3 in3
b) 65 in3 d) 95 in3
16.The base area of a regular pyramid is 200 cm2 find the area of the section
made by a plane bisecting a lateral edge and parallel to the plane of the
base.
a) 100 cm2 c) 50 cm2
b) 200 cm2 d) 150 cm2
17.A regular square pyramid is cut by a plane parallel to the base and bisecting
an edge. If the height of the pyramid is 15 cm and the base edge is 8 cm,
find the volume of the small pyramid cut off.
a) 40 cm3 c) 120 cm3
b) 60 cm3 d) 150 cm3

18.The volume of a pyramid is equal to the volume of the cube of base edge 5
in. is the base area of the pyramid is 25 in2, find its height.
a) 12 in c) 15 in
b) 10 in d) 17 in
19.The total area of a tetrahedron with edge 4 cm is
a) 16√3/2 cm c) 16√3/4 cm
b) 16√3 𝑐𝑚 d) 16√2/2 cm
20.A pyramid and a prism have equal volumes. If the base area of the prism is
2/3 the base area of the pyramid and its height is 12 in, find the height of
the pyramid.
a) 12 c) 20
b) 16 d) 24

Feedback to Post Test


11.C
1. D
12.A
2. A
13.C
3. C
14.B
4. B
15.C
5. C
16.C
6. C
17.A
7. B
18.C
8. D
19.B
9. B
20.D
10.B
MATH 122
SOLID GEOMETRY

UNIT VII
SPHERE
UNIT VII
SPHERES
If you are a very sports minded person, then you must be very familiar with
spheres – all those balls … basketball, volleyball, football, etc. What about
the bid dome where PBA games were once played.

An extension of this study of spheres will be about the spherical zone,


segment, sector and cone. You will find them very familiar as you go
through the text this module.
Carry on. Enjoy the last unit of this module. Feel like its Christmas when you
hang those Christmas balls on your Christmas tree.

The module will expose you to the different solids that can be obtained
from sphere lessons. Their areas and volumes are listed in separate lessons.
You will just as easy as learning the previous solids there will be a pre-test
to check your familiarity with spheres. Otherwise, take on the lessons and
finally you go through the post – test and see if you have mastery of
spheres.
Prerequisite Components
To go through the study of spheres you will need a good foundation of the
previous lessons learned on plane geometry and the previous solids,
respectively:
a) Pythagorean Theorem
b) Circular arcs and sectors
c) Area and circumference of a circle
d) Parallelepipeds, cubes and cylinders
e) Pyramids and cones

General Objectives
At the end of this module you are expected to:

1. Recognize spheres
2. Recognize a spherical cone and spherical segment
3. Recognize a spherical sector and spherical segment
4. Solve problems involving spheres

What to Expect
In this module, you will deal with the following:

1. A pre- test to be able to help you decide if you still have to study every
lesson in this module
2. Check your answers using the key to the pre-test
3. Check the category where you belong by using this table

SCORE VERBAL INTERPRETATIONS


20 Excellent
18-19 Very Good
15-17 Good
12-14 Satisfactory
8-below Needs Improvement
If you score an excellent mark, there is no need to go through this module,
otherwise you need to comprehend every lesson in this module.
4. The lessons
a. The module is divided into three lessons:
Lesson 1: What a sphere is
Lesson 2: Spherical segment and Spherical Zones
Lesson 3: Spherical Sector and Spherical Cones
b. Every lesson will deal with:
i.) Specific Objectives
ii.) Learning Activities
iii.) Practice task
iv.) Feedback to Practice Task
5. A Post Test to help you figure out if you have really learned from this
module.

Pretest
1. Find the area of a sphere whose diameter is 18 cm.
a) 18𝜋𝑐𝑚2
b) 9 𝜋𝑐𝑚2
c) 36 𝜋𝑐𝑚2
d) 324 𝜋𝑐𝑚2
2. Find the volume of a sphere whose radius is 6 cm.

a) 288 𝜋 𝑐𝑚3 c) 48𝜋 𝑐𝑚3


b) 144𝜋 𝑐𝑚3 d) 36𝜋 𝑐𝑚3

3. The ratio of the volume of two spheres is 27:125. Then the ratio of the radii of
the spheres is:
a) 9:5 c) 3:5
b) 3:25 d) 9:25

4. Find the volume of a sphere whose area is 64𝜋.


a) 256𝜋 𝑐𝑚3 c) 256 𝜋/3 cm2
b) 64𝜋 cm2 d) 2048 𝜋 𝑐𝑚2

5. Find the area of a zone on a sphere having a radius of 12 cm if the height of


the zone is 4 cm.
a) 48𝜋 𝑐𝑚2 c) 144𝜋 𝑐𝑚2
b) 96𝜋 𝑐𝑚2 d) 324𝜋 𝑐𝑚2

6. The area of a zone on a sphere is 12 cm2 and the circumference of a great


circle on the sphere is 4 cm. find the altitude of the zone.
a) 2 cm c) 6 cm
b) 3 cm d) 12 cm

7. Find the volume of a spherical segment if the radii of the bases are 4 cm and 5
cm respectively ang its altitude is 3 cm.
a) 20𝜋 cm3 c) 66𝜋 cm 3
b) 33𝜋 cm3 d) 132𝜋 cm3
8. Find the volume of a spherical sector if the altitude of the zone is 18 cm and
the radius of the sphere is 6 cm
a) 72𝜋 𝑐𝑚3 c) 288𝜋 cm3
b) 144𝜋 cm3 d) 432𝜋 cm3

9. Find the area of a spherical segment cut off by a plane 3 cm from the center of
a sphere whose radius is 5 cm.
a) 36𝜋 cm2 c) 56𝜋 cm2
b) 51𝜋 cm2 d) 71𝜋 cm2

10. A tool is in the shape of a spherical cone. Find the volume of the tool if the
area of its zone is 2 cm2 and the radius of its sphere is 30 cm.
a) 10 cm3 c) 30 cm3
b) 20 cm3 d) 40 cm 3

11. What is the radius of the sphere if its surface area is numerically equal to its
volume?
a) 1 c) 4
b) 3 d) 12

12. A cube is 4 cm on an edge. Find the radius of the sphere that can be inscribed
in the cube.
a) 2 cm c) 8 cm
b) 4 cm d) 16 cm

13. If the radius of a sphere is increased by 20%, by what percent is the volume
increased?
a) 20% c) 72.8%
b) 44% d) 89%
14. How far from the center of a sphere of radius 13 cm should a plane be passed
to cut from the sphere a circle having an area of 25𝜋 cm2?
a) 5 cm c) 10 cm
b) 8 cm d) 12 cm

15. If the diameter of a sphere is increased by 20%, by what percent is the area of
the increased?
a) 20% c) 72.8%
b) 44% d) 80%

16. A spherical shell whose outside and inside diameters are 16 cm and 8 cm,
respectively, is melted and molted into a cylinder 24 cm high. Find the
diameter of the cylinder.
a) 5.989 cm c) 9.978 cm
b) 4.989 cm d) 11.978 cm

17. A sphere is inscribed in a frustum of a right circular cone. Find the radius of
the sphere if the diameters of the base of the frustum are 8 cm and 12 cm.
a) 2.484 cm c) 4.988 cm
b) 3.874 cm d) 7.348 cm

18. The area of a zone on a sphere is 14 cm2. The circumference of the great circle
on the sphere is 8 cm. what percent of the area of the sphere is the area of the
zone?
a) 13.985% c) 27.8%
b) 68.72% d) 34.3%
19. An iron sphere just fit into a cube. If the cube is filled with water and when the
sphere is placed in the cube, I kg of water flows out. What is the volume of the
cube?
a) 1079.55 cm3 c) 1420.326 cm3
b) 1954.21 cm 3 d) 1909.854 cm3

20.In order to triple the capacity of a spherical balloon, by what percent must the
surface be increased?
a) 100% c) 208%
b) 130% d) 300%

Feedback to Pre-Test

1) d 6) b 11) b 16) c
2) a 7) c 12) a 17) d
3) c 8) d 13) c 18) b
4) c 9) a 14) d 19) d
5) b 10) b 15) b 20) c
Lesson 1: What a Sphere Is

I. Specific Objects
At the end of this unit, you are expected to:
1. Recognize spheres
2. Identify a sector of a sphere
3. Differentiate great circle from small circle
4. Solve the area and volume of a sphere

II. Learning Activities

A. Discussion

A.1. Definition of a SPHERE

These are spheres:


Have you ever wondered how you generate a spherical surface? All
You need to do is to rotate a semi-circle about its diameter.

A sphere is a solid all of whose points lying on its surface are equidistant from a fixed
point known as the center. The measure of the equal distance from the center to
any point on the surface is the radius.

Small circle
R

Great circle

On a sphere, planes can be made to pass and sections of the sphere


can be obtained. You will note that no matter how a plane is passed
through the sphere, the section produced will always be a circle.

When the radius of the section produced is equal to the radius of the
sphere, a great circle is formed. This can be achieved when you pass the
plane through the center of the sphere. Otherwise, a small circle will be
produced.
A.2. SURFACE AREA of a SPHERE
The surface area, S, of a sphere is always four times the area of
a great circle.

S=4𝜋R2, R= radius of the sphere

B. Illustrative Examples
1. The circumference of a great circle of a sphere is 314.16 cm. find the
area of the sphere,

Solution:
A great circle of a sphere is a section
of the sphere produced when a plane
R passes through the center of the
sphere. Thus, the radius of the great
circle is also the radius of the sphere.

You determine the radius of the sphere from the circumference of the great
circle.
C=2𝜋𝑅
314.16=2 𝜋𝑅
314.16 100
R= =
2(3.1416) 2

R = 50 cm
The area of the sphere is 4𝜋(50)2 = 1000𝜋 cm2
2. Two spheres have radii of 6 inches and 9 inches respectively. What is the
ratio of their areas?

Solution.
You can always compare spheres. If S1 and S2 are the surface areas of
the two spheres whose radii are r1 and r2 respectively, then
𝑆1 4𝜋1^2
=
𝑆2 4𝜋2^2
S1 = r12
S2 r22

6
Substituting S1 = ( )2 = (2/3)2 = 4/9
9
S2

A.3 VOLUMES of SPHERE

The volume of a sphere is 4/3𝜋 times the cube of the radius of the
sphere.

4
V = 𝜋R2
3
C. Illustrative Problem
1. Find the volume of a sphere whose area is 64𝜋

Solution.

You determine the volume of a sphere from the given area.

64𝜋 =4 𝜋R2

16 = R2 R = 4 cm
4
The volume of the sphere is 𝜋(4)3 = 85.333 𝜋 cm3
3

2. A sphere is inscribed in a cylinder 18 cm in diameter. What is the area of


the sphere? What is its volume?

Solution.

When a sphere is inscribed in a cylinder,


their radii are equal. Thus, the radius of
the sphere is 9 cm.

The area of the sphere is

4 𝜋(9)2 = 324 𝜋 cm2

The volume of the sphere is


4
𝜋(9)3 = 972 𝜋 cm3
3
3. A metal sphere just fits into a cube. If the cube is filled with water and
when the sphere is placed in the cube 4500𝜋 𝑐𝑚3 of water flows out. What
is the volume of the cube?

Solution:

When the sphere just fits in the cube, the


e
R diameter of the sphere will equal the edge of

the cube, ie, e = 2R. The volume of the cube is

8R3

To determine the radius of the sphere, consider that the amount of water
which has flown out after the sphere has been put in the cube is the
volume of the sphere. Thus,
4
4500𝜋 = 𝜋𝑅3 R = 15 cm
3

The volume of the cube is 8(15)3 = 27,000 cm3

4. If the radius of a sphere is increased by 10 percent, what is the percent


increase in area? In volume?
Solution:
If you increase the radius R by 10% thus would mean that the new radius is
1.1R. Thus,
The new area is 4𝜋(1.1R) 2 = 1.21𝜋R2 and this would lead to conclude
that the area is increased by 21%
4 4
The new volume is 𝜋(1.1R)3 = 1.331 𝜋R2
3 3
And this means that the new volume is increased by 33.1%.
5. A hallow metal sphere whose outside and inside diameters are 12 cm and 8
cm, respectively is melted and cast into a cylinder 9 cm high. Find the
diameter of the cylinder.

Solution.
r0
9 cm
r1

r
To find the diameter of the cylinder, you must consider that the volume of
the hallow sphere is equal to the volume of the cylinder.
V hollow sphere = V cylinder
4 4 4
𝜋(r0)3 - 𝜋(r1)3 = 𝜋r2h r = √ (216 – 64)
3 3 29
4
(63 – 43) = r2 (9) r = 2/3 √152
3
r = 8.22cm.
The diameter of the cylinder is 16.44 cm.

Practice Task

The world you live is a big sphere. You do not know where it starts
and ends. But right now, you are starting on your last leg of the journey to
end your study
of Math 122. For a starter, why not go through these exercises?

a) Prob. 1, p.85 g) Prob. 15, p.87


b) Prob. 3, p.85 h) Prob. 16, p.88
c) Prob. 4, p.85 i) Prob. 17, p.88
d) Prob. 5, p.85 j) Prob. 18, p.88
e) Prob. 6, p.85 k) Prob. 19, p.88
f) Prob. 14, p.85 l) Prob. 20, p.88

The answers are found on the feedback to Practice Exercise. It will be


heartening to know that you have conquered the world… of sphere, that
is.

III. Feedback to Practice Task

Prob. 1) 230 cm
Prob. 3) 113. 097 cm2
Prob. 4) 24.39 cm
Prob. 5) 2.334 cm
Prob. 6) 3,994.03 kg
Prob. 14) 615.75 cm2; 1,436.755 cm3
Prob. 15) 2.318; 1.517
Prob.16)
Prob.17) 2, 160 cm3
Prob. 18) 188.495 cm3
Prob. 19) 16.2%
Prob. 20) 519,409.99 cm3; 18,350,922 cm2
Lesson 2: Spherical Segment and zone

I. Specific Objectives

At the end of this lesson, you are expected to:


1. Recognize spherical zone and spherical segment
2. Identify the parts of the zone and segment
3. Solve the area of a zone
4. Solve the area and volume of a spherical segment

II. Learning Activities


A. Discussion
A.1 A spherical zone is a portion on the surface of the sphere bounded
by two parallel planes passing through the sphere.

When one of the two planes is tangent to the sphere, a zone of one
base is produced.
R1
h

R2 h

zone of two base zone of one base

To best illustrate a spherical zone, you secure a watermelon and cut the
watermelon into three parts.

} cutting planes

Remove the flesh of the watermelon and all that will be left will be the skin of the
watermelon. These remaining “skin” of the fruit are the zones. Note that you are
able to produce a spherical cone of two bases (middle part) and two spherical
zone of one base (the upper and lower parts.)

A.2 AREA of a ZONE

Since a spherical zone is a surface, then you are only interested in its area.

These are two things you need to know about the zone. First, the radius of
the sphere where the zone was obtained. Second the altitude of the zone which is
the perpendicular distances the parallel planes.
The area of the zone, Z, is 2𝜋 times the product of the radius of the sphere

Z = 2𝜋 Rh

B. Illustrative Examples
1. The altitude of a zone on a sphere having a radius of 12 cm is 5 inches.
Find the area of the zone. The area of the zone is what part of the area
of the sphere?

Solution:

The area of the zone is 2𝜋 (12)(5) = 120 𝜋cm2

The area of the sphere is 4𝜋 (12)2 = 576 𝜋cm2


120 𝜋cm2
The area of the zone is percent of the area of the sphere and this is about
576 𝜋cm2
20.833%

2. The diameter of the sphere is 14 cm; a plane is passed through the


sphere 4 cm from the center. Compute the area of the two parts into
which the sphere is divided.

Solution:

The plane produces two zones of one base. One


zone has an altitude of 3 cm and the other has an
3
altitude of 11 cm. The area of the two parts are:
4
A1 = 2𝜋 (14)(3) = 84 𝜋 cm2
7
A2 = 2𝜋 (14)(11) = 308 𝜋 cm2
A.3 SPHERICAL SEGMENT

A spherical segment is a solid bounded by zone one or two parallel planes.

SEGMENT OF TWO BASES SEGMENT OF ONE BASE

Again you may refer to the watermelon you have just previously cut into three
parts. Without removing the flesh of the watermelon, the three parts are basically
your spherical segments – one of which has two bases and the others have one
base each.

A.4 AREA and VOLUME of a SPHERICAL SEGMENT

Just like in the study of spherical zone, you need to identify the radius of the
sphere where the segment was obtained and the radius/radii of the base/s.

Segment of TWO Segment of ONE


BASES BASE
1. Surface area, Z 2𝜋 Rh 2𝜋 Rh

2. Total area, TA Z + 𝜋r12 + 𝜋r22 Z + 𝜋r2

3. Volume, V 1/6 𝜋h[h2+3r12+ 3r22 1/3 𝜋h2(3R – h)


C. Illustrative Examples

1. Find the volume of a spherical segment cut off by a plane 2 cm from the
center of the sphere whose diameter is 6 cm.

Solution:

The volume of the segments are:


17𝜋
V1 = 1/3 𝜋(1)2[3(3)-1] = cm3
3

325𝜋
V2 = 1/3 𝜋(5)2[3(3)-5] = cm3
3

2. Find the volume of a spherical segment if the radii of the bases are 3 cm
and 5 cm, respectively and the distance between bases is 4 cm.

Solution:

This is a segment of two bases. Thus

V = 1/6 𝜋(4) [42 + 3(3)2 + 3(5)2]

236𝜋
= cm3
3
Practice Task
As you go through these next set of exercises to find out if you have
understood what a zone and a segment of a sphere are, you should
consider having a bite or two of that juicy watermelon! Enjoy these
exercises:

a.) Prob. 7 p.86


b.) Prob. 8 p.86
c.) Prob. 10 p.86
d.) Prob. 11 p.87

How easy did you find these exercises? Find out on the list of answers on
the Feedback to Practice Task.

Feedback to Practice Task

Prob.7) 2010.691 cm2

Prob.8) 8526.675 cm2

Prob.10) 84.823 cm3; 42.943 cm3

Prob.11) 2646.4 kg
Lesson 3: Spherical Sectors and Spherical Cone

I. Specific Objectives

At the end of this lesson, you are expected:

1. Recognize spherical sector and spherical cone


2. Identify the parts of the sector and cone
3. Solve the area and volume of the sector and cone

II. Learning Activities


A. Discussion

A.1 A spherical sector is a solid bounded by a zone and one or two conical
surfaces. The solid is produced when a sector of a circle is revolved about the
axis of the circle.

Sector of a circle to be a spherical sector with two conical


rotated about the axis surfaces
A.2 A spherical cone is a spherical sector with only one conical surface. This can
be achieved when one of the radii of the sector of a circle lies on the axis.

Sector of a circle a spherical cone

SPHERICAL SECTOR/CONE

To define the area and volume of a spherical sector/cone, you need to


define not only the radius of the sphere and the altitude of the zone but also the
radius/radii.

Spherical sector spherical cone

1. Area Z+𝜋r1R+ 𝜋r2R Z+ 𝜋rR


Where: Z= 2𝜋𝑅ℎ
𝜋rR; 𝜋r1R; 𝜋r2R = lateral area of the conical surfaces
1 1
2. Volume ZR ZR
3 3

V= 2/3 𝜋 R2h

Of the conical surface/surfaces produced.


B. Illustrative Examples

1. Find the volume of a spherical sector whose radius is 8 cm and its zone is
formed by a parallel planes 2 and 3 inches respectively from the center of
the sphere and lying on opposite sides of the center.

Solution:
2
The altitude of the zone is 5 cm. thus the volume of the sector is 𝜋(8)2(5) =
3
640
𝜋cm3
3

2. A top has a form of a spherical cone. The slant height is 3 in. and inclined
60° to its axis. What is the volume of the top?

Solution:

Considering the cross-section of the top,

Y= 3cos60° = 1.5 cm

The altitude of the zone is 3-1.5 = 1.5 cm


2
The volume of the top is 𝜋(3)3(1.5) = 9 𝜋cm
3

60°

3 in.
Practice Task
This is the final lap! Victory is in your hands now. Make sure that you are able
to deal with these exercises:

a.) Prob.9 p.86


b.) Prob.12 p.87
c.) Prob.13 p.87

What is the feeling of cutting the ribbon at the finishing line? Great? See for
yourself! Check out the Feedback to Practice Task

Feedback to Practice Task

Prob.9) 70.401%

Prob.12) 8.901 dm3

Prob.13) hollow stem: 2.694m2

Solid stem: 15947.75 cm3


Post Test
1. What is the ratio of the volume of two spheres if the radius of one is twice
the radius of the other?
a) 1:2 c) 1:8
b) 1:4 d) 1:16

2. A steel ball bearing ha a radius of 1 cm. how much steel is needed to make
1000 such ball bearings?
400 2000
a) 𝜋 cm3 c) 𝜋 cm3
3 3
800 4000
b) 𝜋 cm3 d) 𝜋 cm3
3 3

3. The diameter of the earth is about 12,800 km. about 70% of the earth’s
surface is covered by water. How many square kilometers of land are
there?
a) 154 km2 c) 618 km2
b) 360 km2 d) 1441 km2

4. What is the ratio of the surface area of two spheres if the radius of one is
three times the radius of the other?
a) 1:3 c) 1:27
b) 1:9 d) 1:81
5. A dessert requires 15 spherical limes that have 5 cm diameter. You can only
find spherical limes with 7.5 cm diameter. How many limes do you need for
the recipe?
a) 3.5 c) 5.5
b) 4.5 d) 6.5

6. How much leather is there in the cover of a softball that has a


circumference of 12 inches?
a) 14.59 in2 c) 45.84 in2
b) 113.097 in2 d) 144 in2

7. What is the ratio of the area of a great circle of a sphere to the surface area
of the sphere?
a) 1:2 c) 1:8
b) 1:4 d) 1:9

8. Find the volume of a sphere if its great circle has a circumference of 314.16
cm?
a) 10,471.98 cm3 c) 130,899,.69cm3
b) 159,154.94 cm3 d) 523,598.78 cm3

9. If the altitude of a zone of one base is 4 cm and the diameter of the base is
12 cm, find the diameter of the sphere.
a) 2√13 cm c) 9 cm
b) 6.5 cm d)13 cm
10.A tank with a circular base is in the shape of a hemisphere. How much
would it cost to paint the inside of the tank if its radius is 30 ft and each
liter of paint covers 20 ft2 and cost P 250.00?
a) P2356.19 c) P70,685.83
b) P450.39 d) P282,743.33

11. Consider an ice cream cone with a hemisphere of ice cream top. If the
diameter of the top of the cone is 7.62 cm and the height of the cone is
12.7 cm, find the entire surface area of the cone and ice cream.
a) 205.38 cm2 c) 150.28 cm2
b) 158.72 cm2 d) 146.66 cm2

12.Find the volume of the ice cream cone and the ice cream in number 11.
a) 657.58 cm3 c) 206.22 cm3
b) 368.25 cm3 d) 1287.04 cm3

13.What is the surface area of a solid formed by revolving the quadrant of a


circle of diameter 14 cm around the diameter?
a) 14𝜋 cm2 c) 28𝜋 cm2
b) 49𝜋 cm2 d) 98𝜋 cm2

14.What is the volume of the smallest box that will hold a basketball 22 cm in
diameter?
a) 121 cm3 c) 1,331 cm3
b) 484 cm3 d) 10, 648 cm3
15.What is the volume of the smallest right circular cylinder that can hold a
sphere of diameter 14 cm?
a) 1372 𝜋 cm3 c) 343 𝜋 cm3
b) 686 𝜋 cm3 d) 98 𝜋 cm3

16.If S denotes the area of a sphere and V its volume, then


5𝑆 3𝑆
a) V= √4𝜋𝑆 c) V= √4𝜋𝑆
12 4𝜋
b) S= ∛36𝜋𝑉2 d) S=V ∛36𝜋𝑉2

17.What is the weight of a metallic spherical shell 1.75cm thick if the inside
diameter of the shell is 27 cm and if one cubic centimeter of the metal
weighs grams?
a) 17.8 kg c) 36.4 kg
b) 18.2 kg d) 42.8 kg

18.A spherical shell 8 cm thick contains the same amount as a solid sphere
whose radius is 20 cm. find the radius of the outer surface of the shell.
a) 26.2 cm c) 23.52 cm
b) 24 cm d) 22.11cm

19.The altitude of a zone on a sphere having a radius of 10 cm is 6 cm. what


part of the area of the sphere is the area of the zone?
a) 2:3
b) 3:10
c) 1:20
d) 3:20
20.Two spherical cones are made from a circle cut from a piece of heavy
paper. The first is made by cutting along two radii which make an angle o
60° angle to one another. The second is made by cutting along two radii
which make 90° angle to one another. How do their volumes compare?
a) 13.4%
b) 33.4%
c) 50%
d) 66.7%

Feedback to Post Test

17.) C
18.) D
1.) C
19.) B
2.) D
20.) A
3.) A
4.) B
5.) B
6.) C
7.) B
8.) D
9.) D
10.) C

11.) A
12.) C
13.) D
14.) D
15.) B
16.) B

Vous aimerez peut-être aussi